Вы находитесь на странице: 1из 57

American Academy of Orthopaedic Surgeons

2002 Anatomy Self-Assessment Examination


Page 1
1. A patient has right shoulder pain. Figure 1a shows a gadolinium-enhanced
transverse MRI scan at the level of the coracoid. Figure 1b shows an arthroscopic
view of the anterior structures from a posterior portal. These images reveal which
of the following findings?

1- Normal anatomic variant (Buford complex)


2- Glenoid labral tear (superior labrum anterior and posterior)
3- Bankart lesion
4- Avulsion of the biceps tendon
5- Subscapularis tendinitis

PREFERRED RESPONSE: 1

DISCUSSION: The area shown in the arthroscopic view and MRI scan is referred to as a Buford
complex and represents a normal labral variant. It consists of a thickened, cord-like middle
glenohumeral ligament, a superior labral attachment of the middle glenohumeral ligament just
anterior to the biceps tendon, and absence of the anterosuperior labrum. This combination of
findings can be confusing and may simulate labral pathology. Mistaken repair of the lesion back
to the glenoid rim can result in significant loss of external rotation. A Bankart lesion would be
located at the inferior anterior glenoid rim. The subscapularis is seen anterior to the labrum.
Normal variations that occur in the anterosuperior labrum can simulate pathology.

Gusmer PB, Potter HG, Schatz JA, et al: Labral injuries: Accuracy of detection with unenhanced MR imaging of the
shoulder. Radiology 1996;200:519-524.
Griffin LY (ed): Orthopaedic Knowledge Update: Sports Medicine. Rosemont, IL, American Academy of Orthopaedic
Surgeons, 1994, pp 47-63.
Williams MM, Snyder SJ, Buford D Jr: The Buford complex: The cord-like middle glenohumeral ligament and absent
anterosuperior labrum complex. A normal anatomic capsulolabral variant. Arthroscopy 1994;10:241-247.

2. What muscle attaches to the site


shown by the arrow in Figure 2?

1- Teres minor
2- Teres major

Dr. Ahmed Altaei


American Academy of Orthopaedic Surgeons
2002 Anatomy Self-Assessment Examination
Page 2
3- Latissimus dorsi
4- Pectoralis major
5- Pectoralis minor

PREFERRED RESPONSE: 3

DISCUSSION: The latissimus dorsi inserts on the humerus metaphysis between the pectoralis
major (posterior) and teres major (anterior). Teres minor inserts on the base of the greater
tuberosity. Pectoralis minor does not insert on the humerus.

Williams PL, Warwick R, Dyson M, Bannister LH: Neurology, in Grays Anatomy, ed 37. Edinburgh, Scotland, Churchill
Livingstone, 1989, pp 1131-1132.
Last RJ: Anatomy: Regional and Applied, ed 6. London, England, Churchill Livingstone, 1978, pp 131-132.

3. Figures 3a and 3b show the inversion stress radiographs of a patients ankle. What
is the most likely ligament injury pattern?

1- Calcaneofibular alone
2- Posterior talofibular alone
3- Posterior talofibular and deltoid
4- Anterior talofibular and deltoid
5- Anterior talofibular and
calcaneofibular

PREFERRED RESPONSE: 5

DISCUSSION: The radiographic findings show 30 degrees of talar tilt (severe) and 10 mm of
anterior translation that typically involves laxity of both of the major lateral ligaments of the
ankle (anterior talofibular and calcaneofibular). There is no evidence of deltoid laxity.

Harper MC: Stress radiographs in the diagnosis of lateral instability of the ankle and hindfoot. Foot Ankle 1992;13:435-
438.
Mizel MS, Miller RA, Scioli MW (eds): Orthopaedic Knowledge Update Foot and Ankle 2. Rosemont, IL, American
Academy of Orthopaedic Surgeons, 1998, pp 39-54.

4. Posterior sternoclavicular dislocations are most commonly associated with which of


the following complications?

1- Chronic instability
2- Brachial plexus palsy

Dr. Ahmed Altaei


American Academy of Orthopaedic Surgeons
2002 Anatomy Self-Assessment Examination
Page 3
3- Pneumothorax
4- Esophageal compression
5- Tracheal compression

PREFERRED RESPONSE: 5

DISCUSSION: Posterior sternoclavicular dislocations are commonly associated with tracheal


compression, which can be a life-threatening condition requiring immediate reduction. The other
listed complications are less common.

Brooks AL, Henning GD: Injury to the proximal clavicular epiphysis, abstracted. J Bone Joint Surg Am 1972;54:1347-
1348.
Beaty JH (ed): Orthopaedic Knowledge Update 6. Rosemont, IL, American Academy of Orthopaedic Surgeons, 1999, pp
255-260.

5. An AP radiograph of the pelvis is shown in Figure 4. What muscle attaches to the


avulsed fragment of bone identified by the arrow?

1- Short head of the biceps femoris


2- Adductor longus
3- Pectineus
4- Piriformis
5- Semitendinosus

PREFERRED RESPONSE: 5

DISCUSSION: The radiograph reveals an avulsion of the ischial apophysis, most likely the
result of violent contraction of the attached hamstring tendons (semimembranosus,
semitendinosus, and long head of the biceps femoris). The short head of the biceps femoris
arises from the linea aspera on the posterior femur. The pectineus and adductor longus attach to
the pubic portion of the pelvis. The piriformis runs from the sacrum to the femur.

Woodburne RT (ed): Essentials of Human Anatomy. New York, NY, Oxford University Press, 1978, pp 542-545.
Metzmaker JN, Pappas AM: Avulsion fractures of the pelvis. Am J Sports Med 1985;13:349-358.

6. A patient with an acromioclavicular dislocation has a very prominent distal clavicle.


Examination reveals that the deformity increases rather than reduces with an
isometric shoulder shrug. Which of the following structures is most likely intact?

1- Trapezoid ligament
2- Conoid ligament
3- Acromioclavicular ligament
4- Deltoid muscle origin

Dr. Ahmed Altaei


American Academy of Orthopaedic Surgeons
2002 Anatomy Self-Assessment Examination
Page 4
5- Trapezius muscle insertion

PREFERRED RESPONSE: 5

DISCUSSION: Severely displaced acromioclavicular injuries disrupt the deltotrapezial fascia


and muscular origin in addition to the ligaments (acromioclavicular and coracoclavicular or
trapezoid and conoid). When the deltoid is still attached to the clavicle, an isometric shoulder
shrug will tend to reduce the displacement. When the deltoid is detached but the trapezius is
attached, this manuever will increase the deformity and surgery may be indicated.

Weinstein DM, McCann PD, McIlveen SJ, Flatow EL, Bigliani LU: Surgical treatment of complete acromioclavicular
dislocations. Am J Sports Med 1995;23:324-331.

7. Figures 5a and 5b show axial and coronal MRI images of the left ankle of a patient
with lateral ankle pain. What is the most likely diagnosis?

1- Peroneus brevis tendon tear


2- Posterior tibialis tendon tear
3- Talar dome osteochondral loose body
4- Talar fracture
5- Flexor hallucis tenosynovitis

PREFERRED RESPONSE: 1

DISCUSSION: The figures show a longitudinal split within the peroneus brevis tendon as it
courses posterior to the fibula. The peroneus longus tendon has been driven between the medial
and lateral components of the peroneus brevis tendon. Peroneal split syndrome is a cause of
lateral ankle pain but may be less asymptomatic in the elderly. It may be associated with tendon
subluxation following a tear of the superior peroneal retinaculum.

Mink JH: Tendons, in Deutsch AL, Mink JH, Kerr R (eds): MRI of the Foot and Ankle. New York, NY, Raven Press, 1992,
pp 135-172.

Dr. Ahmed Altaei


American Academy of Orthopaedic Surgeons
2002 Anatomy Self-Assessment Examination
Page 5
8. Which of the following anatomic structures is often difficult to visualize during
elbow arthroscopy?

1- Ulnar collateral ligament


2- Olecranon fossa
3- Radial head
4- Coronoid process
5- Anterior superior capsular attachment to the humerus

PREFERRED RESPONSE: 1

DISCUSSION: The ulnar collateral ligament is often difficult to visualize during elbow
arthroscopy. It can be seen clearly in only 10% to 30% of elbow arthroscopies. All of the other
structures should be easily and thoroughly seen and palpated during elbow arthroscopy.

Johnson LL: Arthroscopic Surgery: Principles and Practice. St Louis, MO, CV Mosby, 1988.
Morrey BF: Arthroscopy of the elbow. Instr Course Lect 1986;35:102-107.

9. The quadrilateral space in the shoulder contains which of the following structures?

1- Axillary nerve and posterior humeral circumflex artery


2- Axillary artery and radial nerve
3- Axillary artery and axillary nerve
4- Recurrent suprascapular nerve and artery
5- Profunda brachii artery

PREFERRED RESPONSE: 1

DISCUSSION: The quadrilateral or quadrangular space of the shoulder is formed laterally by the
humerus, proximally by the subscapularis (and teres minor viewed from posterior), distally by
the teres major, and medially by the long head of triceps. The posterior humeral circumflex
artery and axillary nerve pass through it. The axillary artery is more proximal. The radial nerve
and profunda brachii pass through a triangular space more inferior. The circumflex scapular
artery passes through a triangular space more medial.

Hollinshead WH: Textbook of Anatomy, ed 3. Hagerstown, MD, Harper and Row, 1974, pp 205-206.
Last RJ: Anatomy: Regional and Applied, ed 6. London, England, Churchill Livingstone, 1978, pp 61-62.

10. Based on the MRI scan shown in


Figure 6, the abnormal signal is seen
in what carpal bone?

1- Triquetrum
2- Lunate
3- Hamate
4- Trapezium
Dr. Ahmed Altaei
American Academy of Orthopaedic Surgeons
2002 Anatomy Self-Assessment Examination
Page 6
5- Trapezoid

PREFERRED RESPONSE: 5

DISCUSSION: The MRI scan reveals an abnormal signal in the trapezoid, which lies adjacent to
the capitate in the distal carpal row. The tumor is a giant cell tumor of bone.

Cooney WP, Linscheid RL, Dobyns JH: The Wrist: Diagnosis and Operative Treatment. St Louis, MO, Mosby-Year Book,
1998, vol 1, pp 278-282.
Green DP, Hotchkiss RN, Pederson WC (eds): Operative Hand Surgery, ed 4. New York, NY, Churchill Livingstone, 1999,
pp 2238-2240.

11. The recurrent motor branch of the median nerve innervates which of the following
muscles?

1- Abductor pollicis brevis, first dorsal interosseous, opponens pollicis


2- Abductor pollicis brevis, flexor pollicis brevis, opponens pollicis
3- Adductor pollicis, first dorsal interosseous, opponens pollicis
4- Adductor pollicis, flexor pollicis brevis (deep and superficial heads)
5- Adductor pollicis, flexor pollicis brevis, opponens pollicis

PREFERRED RESPONSE: 2

DISCUSSION: The recurrent motor branch of the median nerve supplies the thenar muscles
(abductor pollicis brevis, flexor pollicis brevis, and opponens pollicis) that are primarily
responsible for thumb opposition. The nerve can be injured in carpal tunnel release. A branch of
the nerve also supplies the first lumbrical. The adductor pollicis and the interossei are supplied
by the ulnar nerve.

Last RJ: Anatomy: Regional and Applied, ed 6. London, England, Churchill Livingstone, 1978, p 109.
Hoppenfeld S, deBoer P: Surgical Exposures in Orthopaedics: The Anatomic Approach. Philadelphia, PA, JB Lippincott,
1984, p 170.

12. Which of the following nerves innervates the


muscle that originates from the middle third of the
dorsal surface of the lateral border of the scapula,
as shown in Figure 7?

1- Musculocutaneous
2- Lower subscapular
3- Axillary
4- Suprascapular
5- Infrascapular

PREFERRED RESPONSE: 3

Dr. Ahmed Altaei


American Academy of Orthopaedic Surgeons
2002 Anatomy Self-Assessment Examination
Page 7

DISCUSSION: Teres minor originates from the middle third of the dorsal surface of the lateral
border of the scapula. It is supplied by the axillary nerve (C5).

Williams PL, Warwick R, Dyson M, Bannister LH: Myology, in Grays Anatomy, ed 37. Edinburgh, Scotland, Churchill
Livingstone, 1989, pp 611-615.
Last RJ: Anatomy: Regional and Applied, ed 6. London, England, Churchill Livingstone, 1978, pp 68-72.

13. Based on the MR arthrogram of the elbow shown in Figure 8, which of the following
structures is torn?

1- Common flexor tendon


2- Anconeus
3- Radial collateral ligament
4- Ulnar collateral ligament
5- Lateral ulnar collateral ligament

PREFERRED RESPONSE: 4

DISCUSSION: Based on the MR arthrogram in which gadolinium (bright on T1-weighted


images) was injected into the joint space prior to imaging, the study shows a tear of the anterior
band of the ulnar collateral ligament (UCL). The disruption in the distal end of the UCL is
outlined by contrast. A small collection of contrast extravasation into the flexor musculature
further confirms the presence of a tear. The UCL has a broad-based attachment on the medial
epicondyle and has a pointed or tapered attachment distally on the ulna. Most UCL tears occur
distally at the ulnar (coronoid) attachment.
MR arthrography provides improved sensitivity compared to conventional MRI, without
contrast, for the detection of UCL pathology, particularly in the subacute or chronic setting.
After the soft-tissue edema and joint fluid associated with the injury have resolved, the torn end
of the ligament may lie in contact with its adjacent attachment and create a false-negative
appearance. In this patient, a noncontrasted MR arthrogram showed no tear, yet the tear is
apparent with intra-articular contrast and distention. MR arthrography of the elbow also may be
useful in detecting intra-articular bodies or in evaluation for loose osteochondral fragments or
flaps.

Morrey BF: Acute and chronic instability of the elbow. J Am Acad Orthop Surg 1996;4:117-128.
Resnick D, Kang HS (eds): Internal Derangements of Joints: Emphasis on MR Imaging. Philadelphia, PA, WB Saunders,
1997, pp 200-210.

Dr. Ahmed Altaei


American Academy of Orthopaedic Surgeons
2002 Anatomy Self-Assessment Examination
Page 8
14. A 26-year-old man has recurrent right knee pain. Figures 9a and 9b show
consecutive sagittal T2-weighted MRI scans, and Figure 9c shows a coronal T1-
weighted MRI scan. What is the most likely diagnosis?

1- Bucket-handle tear of the lateral


meniscus
2- Medial meniscus tear
3- Discoid lateral meniscus
4- Posterior cruciate ligament tear
5- Normal MRI of the knee

PREFERRED RESPONSE: 3

DISCUSSION: A discoid meniscus is a large disk-like meniscus. It is seen in the lateral


meniscus in 3% of the population; a discoid medial meniscus is much less common. It can be
identified on the coronal view by noting meniscal tissue extending into the tibial spine at the
intercondylar notch.
The average width of a normal meniscus is less than 11 mm. A bow-tie appearance should not
be seen on more than two consecutive sagittal images because the conventional thickness of the
sagittal slices is 3 mm and the interval between two consecutive slices is 1.5 mm. Two sagittal
slices will cover a 9-mm thickness. A discoid meniscus can be diagnosed on the sagittal views
by noting a bow-tie appearance on more than two consecutive images.

Helms CA: MR image of the knee, in Fundamentals of Skeletal Radiology, ed 2. Philadelphia, PA, WB Saunders, 1995, pp
172-191.
Mink JH, Deutsch AL: The knee, in MRI of the Musculoskeletal System, ed 1. New York, NY, Raven Press, 1990, pp 251-
387.

Dr. Ahmed Altaei


American Academy of Orthopaedic Surgeons
2002 Anatomy Self-Assessment Examination
Page 9
15. The gluteus maximus is innervated by which of the following nerves?

1- Superior gluteal
2- Inferior gluteal
3- Pudendal
4- Anterior division of the femoral
5- Posterior division of the femoral

PREFERRED RESPONSE: 2

DISCUSSION: The inferior gluteal nerve supplies the gluteus maximus muscle. The superior
gluteal nerve supplies the gluteus medius, gluteus minimus, and tensor fascia lata muscles. The
femoral nerve supplies the quadriceps, sartorius, and pectineus muscles. The pudendal nerve is
primarily a sensory nerve.

Last RJ: Anatomy: Regional and Applied, ed 6. London, England, Churchill Livingstone, 1978, pp 146-147.

16. The dorsal (Thompson) approach to the proximal forearm uses which of the
following intermuscular intervals?

1- Extensor carpi radialis longus and the extensor carpi radialis brevis
2- Extensor pollicis longus and the extensor pollicis brevis
3- Extensor digitorum communis and the extensor carpi radialis brevis
4- Extensor carpi ulnaris and the extensor carpi radialis brevis
5- Abductor pollicis longus and the extensor carpi radialis brevis

PREFERRED RESPONSE: 3

DISCUSSION: The Thompson posterior approach is used in treatment of fractures of the


proximal radius. Dissection is carried out through the interval between the extensor carpi
radialis brevis (radial nerve) and the extensor digitorum communis (posterior interosseous
nerve). To identify this interval, the forearm is pronated and the mobile lateral wad of muscles
(the ulnar-most belly is the extensor carpi radialis brevis) is grasped with the thumb and finger
and pulled from the much less mobile mass of the extensor digitorum communis. The furrow
created is marked with a skin marker for subsequent skin incision. The skin incision follows a
line from the lateral epicondyle of the humerus to a point corresponding to the middle of the
posterior aspect of the wrist. Distally, the intermuscular plane is between the extensor carpi
radialis brevis and the extensor pollicis longus.

Crenshaw AH Jr: Surgical techniques and approaches, in Canale ST (ed): Campbells Operative Orthopaedics, ed 9. St
Louis, MO, Mosby-Year Book, 1998, vol 1, pp 128-129.
Hoppenfeld S, deBoer P: Posterior approach to the radius, in Surgical Exposures in Orthopaedics: The Anatomic Approach,
ed 2. Philadelphia, PA, Lippincott-Raven, 1992, pp 136-146.
Thompson JE: Anatomical methods of approach in operations on the long bones of the extremities. Ann Surg 1918;68:309-
316.

Dr. Ahmed Altaei


American Academy of Orthopaedic Surgeons
2002 Anatomy Self-Assessment Examination
Page 10
17. A 45-year-old man who smokes reports the rapid onset of color changes and
coolness in the fingers. Examination shows an abnormal Allen test. Plain
radiographs of the hand and wrist are normal. Which of the following studies will
best aid in diagnosis?

1- Contrast CT of the hand and wrist


2- MRI of the hand and wrist
3- Contrast angiography of the involved upper extremity
4- Digital subtraction angiography
5- Single-shot fluoroscopic angiography

PREFERRED RESPONSE: 3

DISCUSSION: The patient has symptoms typical of Raynauds phenomenon secondary to


underlying vascular disease. The next most appropriate step in the management of this patient
should be to perform contrast angiography on the involved upper extremity to look for proximal
or distal arterial lesions or insufficiencies. MRI and contrast CT are not as specific as
angiography for the identification of vascular lesions of the upper extremity. Although patients
with primary Raynauds vasospastic disease can have normal angiographic findings, they
typically are younger than age 40 years, are female, and have normal results on an Allen test.

Green DP, Hotchkiss RN, Pederson WC (eds): Operative Hand Surgery, ed 4. New York, NY, Churchill Livingstone, 1999,
pp 2288-2290.
Manske PR (ed): Hand Surgery Update. Rosemont, IL, American Society for Surgery of the Hand, 1994, pp 197-205.

18. A purulent flexor tenosynovitis of the thumb may communicate with the small
finger flexor through which of the following structures?

1- Hypothenar space
2- Thenar space
3- Midpalmar space
4- Distal forearm (Paronas space)
5- Lumbrical canal

PREFERRED RESPONSE: 4

DISCUSSION: Only the flexor sheaths of the thumb and small finger are continuous from the
digit through the carpal canal and into the distal forearm. If one of the sheaths ruptures from
synovitis, it may contaminate the other sheath through Paronas space in the distal forearm. This
potential space lies superficial to the pronator quadratus and deep to the flexor tendons.

Green DP, Hotchkiss RN, Pederson WC (eds): Operative Hand Surgery, ed 4. New York, NY, Churchill Livingstone, 1999,
pp 1044-1045.
Burkhalter WE: Deep space infections. Hand Clin 1989;5:553-559.

Dr. Ahmed Altaei


American Academy of Orthopaedic Surgeons
2002 Anatomy Self-Assessment Examination
Page 11
19. Which of the following nerves travels with the deep palmar arch?

1- Recurrent motor branch of the median nerve


2- Medial branch of the median nerve
3- Lateral branch of the median nerve
4- Superficial branch of the ulnar nerve
5- Deep motor branch of the ulnar nerve

PREFERRED RESPONSE: 5

DISCUSSION: The ulnar nerve divides alongside the pisiform, and the deep branch supplies the
three hypothenar muscles and crosses the palm with the deep palmar arch to supply the two ulnar
lumbricals, all interossei, and finally the adductor pollicis. The superficial branch supplies the
ulnar digital branches to the small and ring fingers. The median nerve branches are more
superficial in the palm near the superficial palmar arch.

Last RJ: Anatomy: Regional and Applied, ed 6. London, England, Churchill Livingstone, 1978, p 109.
Hoppenfeld S, deBoer P: Surgical Exposures in Orthopaedics: The Anatomic Approach. Philadelphia, PA, JB Lippincott,
1984, pp 166-169.

20. Figures 10a through 10c show the plain radiograph and MRI scans of a 41-year-old
man who has right hip pain. What is the most likely diagnosis?

1- Giant cell tumor


2- Rheumatoid arthritis
3- Metastatic carcinoma
4- Osteonecrosis
5- Transient osteoporosis

PREFERRED RESPONSE: 5

Dr. Ahmed Altaei


American Academy of Orthopaedic Surgeons
2002 Anatomy Self-Assessment Examination
Page 12

DISCUSSION: Transient osteoporosis is a self-limited painful but reversible disorder. Although


first described in pregnant women, it is more common in young to middle-aged men. The
radiograph shows loss of mineralization in the right hip relative to the left side. There is no
osseous destruction or cortical expansion typical of metastasis or giant cell tumor. The process is
confined to the femoral side of the joint unlike rheumatoid arthritis, which would be centered in
the joint. Osteonecrosis is better defined with sharp but irregularly shaped margins, and there is
no double-line sign. The MRI scans reveal diffuse edema in the femoral head and neck that is
atypical for osteonecrosis. Transient osteoporosis may recur in the same or opposite hip.

El-Khoury G: MRI of the Musculoskeletal System. Philadelphia, PA, JB Lippincott, 1998, p 241.

21. Figure 11 shows the anatomic dissection of the medial side of the knee joint after
removal of the superficial fascia. The arrow is pointing to what structure?

1- Semitendinosus tendon
2- Gracilis tendon
3- Sartorius tendon
4- Semimembranosus tendon
5- Medial collateral ligament

PREFERRED RESPONSE: 1

DISCUSSION: The semitendinosus and gracilis tendons lie beneath the superficial fascia and
superficial to the medial collateral ligament. The semitendinosus is located more inferior to the
gracilis tendon. The sartorius is more posterior and distal as is the medial collateral ligament.
The semimembranosus is posterior.

Pagnani MJ, Warner JJ, OBrien SJ, Warren RF: Anatomic considerations in harvesting the semitendinosus and gracilis
tendons and a technique of harvest. Am J Sports Med 1993;21:565-571.
Warren LF, Marshall JL: The supporting structures and layers on the medial side of the knee: An anatomical analysis. J
Bone Joint Surg Am 1979;61:56-62.

22. Figure 12 shows a lateral radiograph of the elbow.


What is the most likely diagnosis?

1- Osteochondroma
2- Old fracture fragment
3- Heterotopic ossification
4- Normal anatomic variant
5- Osteosarcoma
Dr. Ahmed Altaei
American Academy of Orthopaedic Surgeons
2002 Anatomy Self-Assessment Examination
Page 13

PREFERRED RESPONSE: 4

DISCUSSION: The figure shows a supracondylar process, which is a normal anatomic variant.
An osteochondroma tends to occur more toward the end of bones, and the medullary space of the
underlying bone extends into the base of the osteochondroma. The presence of a supracondylar
process is usually asymptomatic. However, the ligament of Struthers that always extends from
the supracondylar process to the medial epicondyle can result in median nerve entrapment
secondary to trauma.

Last RJ: Anatomy: Regional and Applied, ed 6. London, England, Churchill Livingstone, 1978, pp 132-133.
Keats TE, Anderson MW: Atlas of Normal Roentgen Variants That May Simulate Disease, ed 7. St Louis, MO, Mosby Inc,
2001, p 497.

23. Which of the following nerves is most likely responsible for symptoms associated
with plantar fasciitis?

1- Medial plantar
2- Medial calcaneal
3- First branch of lateral plantar
4- Lateral plantar
5- Lateral calcaneal

PREFERRED RESPONSE: 5

DISCUSSION: The first branch of the lateral calcaneal nerve innervates the abductor digiti
minimi. It is reported to be trapped at the interval between the abductor hallucis and the
quadratus plantae muscles.

Baxter DE, Pfeffer GB, Thigpen M: Chronic heel pain: Treatment rationale. Orthop Clin North Am 1989;20:563-569.

24. A 16-year-old cheerleader reports an ache in the right shoulder and arm that is
worse after activity. She denies any history of acute trauma. Examination reveals a
positive sulcus sign and an AP glide test with a posterior and anterior apprehension
sign. To confirm a diagnosis of multidirectional instability, which of the following
imaging studies is most appropriate?

1- Ultrasound
2- CT
3- Bone scan
4- Scapular Y-view of the shoulder
5- Stress views of the shoulder

PREFERRED RESPONSE: 5

Dr. Ahmed Altaei


American Academy of Orthopaedic Surgeons
2002 Anatomy Self-Assessment Examination
Page 14

DISCUSSION: Multidirectional instability is a common finding in young female athletes. The


anatomic structures are all intact but are hypermobile; therefore, CT and bone scans and scapular
Y-views are often normal. Obtaining a weighted or AP stress view while applying downward
traction on the arm will document instability and hypermobility of the joint. MRI generally is
not indicated in this condition. Ultrasound is used primarily for rotator cuff pathology.

Neer CS II, Foster CR: Inferior capsular shift for involuntary inferior and multidirectional instability of the shoulder: A
preliminary report. J Bone Joint Surg Am 1980;62:897-908.
Warner JJ, Johnson D, Miller M, Caborn DN: Technique for selecting capsular tightness in repair of anterior-inferior
shoulder instability. J Shoulder Elbow Surg 1995;4:352-364.

25. Which of the following findings is seen in the chest radiograph shown in Figure 13?

1- Fracture-dislocation at T2-3
2- Sternoclavicular fracture-
dislocation
3- Proximal humerus fracture
4- Pneumothorax
5- Distal clavicle osteolysis

PREFERRED RESPONSE: 3

DISCUSSION: Orthopaedic surgeons are often responsible for interpreting radiographs of


general examinations such as the chest radiograph shown. For accurate interpretation, it is
important to systematically review all of the information available on the radiograph. Using this
approach, the fracture of the left proximal humerus is readily recognized. Linear air soft-tissue
density at the lung periphery would suggest a pneumothorax, but this finding is not shown on the
radiograph. The upper thoracic spine is well aligned. The sternoclavicular and distal clavicles
are normal.

Bone LB: Emergency treatment of the injured patient, in Browner BD, Jupiter JB, Levine AM, Trafton PG (eds): Skeletal
Trauma. Philadelphia, PA, WB Saunders, 1992, pp 133-138.

26. Figure 14 shows a lateral radiograph of a knee


joint. The bony structure indicated by the
arrow is a sesamoid bone that resides in what
tendon?

1- Biceps femoris
2- Lateral gastrocnemius
3- Semimembranosus
4- Semitendinosus
5- Popliteus

Dr. Ahmed Altaei


American Academy of Orthopaedic Surgeons
2002 Anatomy Self-Assessment Examination
Page 15
PREFERRED RESPONSE: 2

DISCUSSION: The radiograph shows a fabella, a sesamoid bone that is usually found within the
tendon of the lateral head of the gastrocnemius. It can be confused with a loose body on
radiographs. It occurs in 18% of patients and is often bilateral.

Anderson JE (ed): Grants Atlas of Anatomy, ed 7. Baltimore, MD, Williams & Wilkins, 1978, pp 4-69.
Weissman BNW, Sledge CB (ed): Orthopaedic Radiology. Philadelphia, PA, WB Saunders, 1986, p 514.

27. Talar compression syndrome in ballet dancers typically involves injury to which of
the following structures?

1- Sustentaculum tali
2- Lateral process of the talus
3- Posterior process of the calcaneus
4- Os tibialis externum
5- Os trigonum

PREFERRED RESPONSE: 5

DISCUSSION: Talar compression syndrome is also known as os trigonum syndrome or posterior


ankle impingement syndrome and occurs in activities involving extreme ankle plantar flexion. It
involves pinching of the posterior talus (os trigonum or posterior process of the talus) between
the calcaneus and tibia. The flexor hallucis longus also may be impinged. The other structures
are not commonly injured in this syndrome.

Brodsky AE, Khalil MA: Talar compression syndrome. Am J Sports Med 1986;14:472-476.
Wredmark T, Carlstedt CA, Bauer H, Saartok T: Os trigonum syndrome: A clinical entity in ballet dancers. Foot Ankle
1991;11:404-406.
Marotta JJ, Micheli LJ: Os trigonum impingement in dancers. Am J Sports Med 1992;20:533-536.

28. The sartorius muscle is innervated by which of the following nerves?

1- Femoral
2- Obturator
3- Superior gluteal
4- Inferior gluteal
5- Ilioinguinal

PREFERRED RESPONSE: 1

DISCUSSION: The femoral nerve enters the thigh behind the inguinal ligament, lying on the
surface of the iliopsoas muscle lateral to the femoral artery and vein. The nerve divides into
numerous muscular and cutaneous branches in the femoral triangle. The first motor branch
(sometimes two branches) is to the sartorius. There is a variable branch to the pectineus.
Subsequent branches go to the rectus femoris and then the vastus muscles in variable order. The

Dr. Ahmed Altaei


American Academy of Orthopaedic Surgeons
2002 Anatomy Self-Assessment Examination
Page 16
last motor branch is to the articularis genu. The muscular branches can be injured in anterior
approaches to the hip, especially the middle window of the ilioinguinal approach.

Hollinshead WH: Textbook of Anatomy, ed 3. Hagerstown, MD, Harper and Row, 1974, p 404.
Last RJ: Anatomy: Regional and Applied, ed 6. London, England, Churchill Livingstone, 1978, p 139.

29. Pacinian corpuscles are lamellated nerve endings that are responsible for providing
the perception of

1- pain.
2- light touch.
3- pressure.
4- temperature.
5- vibration.

PREFERRED RESPONSE: 3

DISCUSSION: Pacinian corpuscles are nerve endings that provide the perception of pressure.

Sunderland SS: Nerves and Nerve Injuries, ed 2. New York, NY, Churchill Livingstone, 1978, pp 343-347.

30. An elite gymnast injured her ankle in an awkward dismount 36 hours ago.
Examination reveals weakness on single leg step-up. A clinical photograph of the
medial ankle is shown in Figure 15. Plain radiographs are normal. To help confirm
the diagnosis, the next step in evaluation should consist of

1- MRI to assess for posterior tibialis tendon damage.


2- enhanced CT to assess for an osteochondral fracture.
3- tenography of the peroneal tendons to assess for
tendon sheath tearing with subluxation.
4- a bone scan to assess for occult fracture.
5- arthroscopy of the ankle to assess for articular
cartilage damage.

PREFERRED RESPONSE: 1

DISCUSSION: Ecchymosis on the medial side of the ankle is distributed in the posterior tibialis
tendon sheath location, posterior to the medial malleolus, and extending inferiorly to the
tendons attachment on the navicular. MRI is the imaging study of choice to determine the
extent of tendon damage. MRI will also help assess the deltoid ligament. Bone scans and CT
are helpful in identifying osteochondral fractures and occult fractures; however, these studies are
not indicated for this patient. Peroneal tendons are located lateral on the ankle. Arthroscopy of
the ankle joint would not be helpful in assessing the posterior tibial tendons.

Dr. Ahmed Altaei


American Academy of Orthopaedic Surgeons
2002 Anatomy Self-Assessment Examination
Page 17

Lutter LD, Mizel MS, Pfeffer GB (eds): Orthopaedic Knowledge Update: Foot and Ankle. Rosemont, IL, American
Academy of Orthopaedic Surgeons, 1994, pp 307-317.
Rosenberg ZS, Cheung Y, Jahss MH, Noto AM, Norman A, Leeds NE: Rupture of the posterior tibial tendon: CT and MR
imaging with surgical correlation. Radiology 1988;169:229-235.

31. Figure 16 shows an axial MRI scan through the knee joint. What structure is
identified by the arrow?

1- Anterior cruciate ligament


2- Posterior cruciate ligament
3- Ligament of Wrisberg
4- Ligamentum mucosum
5- Popliteus tendon

PREFERRED RESPONSE: 1

DISCUSSION: The anterior cruciate ligament can be visualized on an axial MRI scan as a low-
signal structure lying in the lateral aspect of the intercondylar notch. Visualization in multiple
planes increases the accuracy of MRI to view the anterior cruciate ligament. The posterior
cruciate ligament and ligament of Wrisberg are located on the medial wall of the notch. The
ligamentum mucosum is anterior to the notch, and the popliteus tendon is posterior to the lateral
femoral condyle.

Resnick D, Kang HS (eds): Internal Derangements of Joints: Emphasis on MR Imaging. Philadelphia, PA, WB Saunders,
1997, pp 675-699.
Fitzgerald SW, Remer EM, Friedman H, Rogers LF, Hendrix RW, Schafer MF: MR evaluation of the anterior cruciate
ligament: Value of supplementing sagittal images with coronal and axial images. Am J Roentgenol 1993;160:1233-1237.

32. Which of the following nerves is most commonly injured when obtaining a bone
graft from the posterior ilium?

1- Lateral femoral cutaneous


2- Superior gluteal
3- Cluneal
4- L5 nerve root
5- S1 nerve root

PREFERRED RESPONSE: 3

Dr. Ahmed Altaei


American Academy of Orthopaedic Surgeons
2002 Anatomy Self-Assessment Examination
Page 18
DISCUSSION: Cutaneous sensation to the buttock is provided by the superior, middle, and
inferior cluneal nerves. The superior cluneal nerves are the lateral branches of the dorsal rami of
the upper three lumbar nerves and penetrate deep fascia just proximal to the iliac crest. They
pass distally to the skin of the buttock and will be injured if the exposure extends more than 8 cm
anterolateral to the posterior superior iliac spine. The lateral femoral cutaneous nerve can be
injured in an anterior ilium bone graft. The superior gluteal nerve or even the sciatic nerve can
be injured if bone is removed from the sciatic notch or dissection is not kept subperiosteal;
however, the rate of injury is far less than cluneal nerve injury. The L5 and S1 nerve roots are
anterior and can be injured if the inner table bone is harvested and the dissection is not kept
subperiosteal or is too medial; however, the rate of injury still is far less than cluneal nerve
injury.

Hoppenfeld S, deBoer P: Surgical Exposures in Orthopaedics: The Anatomic Approach. Philadelphia, PA, JB Lippincott,
1984, pp 295-297.
Hollinshead WH: Textbook of Anatomy, ed 3. Hagerstown, MD, Harper and Row, 1974, p 379.
Last RJ: Anatomy: Regional and Applied, ed 6. London, England, Churchill Livingstone, 1978, p 23.
Ebraheim NA, Elgafy H, Xu R: Bone-graft harvesting from iliac and fibular donor sites: Techniques and complications. J
Am Acad Orthop Surg 2001;9:210-218.

33. Based on the findings seen in the posteroanterior radiograph of the wrist shown in
Figure 17, which of the following structures is torn?

1- Distal radioulnar ligament


2- Radioscapholunate ligament
3- Radioscaphocapitate ligament
4- Scapholunate interosseous ligament
5- Triangular fibrocartilage

PREFERRED RESPONSE: 4

DISCUSSION: The radiograph shows widening between the scaphoid and lunate. The normal
variance is up to 5 mm. Although several ligaments may be torn, the scapholunate interosseous
ligament must be torn for this widening to occur.

Cooney WP, Linscheid RL, Dobyns JH: The Wrist: Diagnosis and Operative Treatment. St Louis, MO, Mosby-Year Book,
1998, vol 1, pp 503-506.
Green DP, Hotchkiss RN, Pederson WC (eds): Operative Hand Surgery, ed 4. New York, NY, Churchill Livingstone, 1999,
pp 884-885.

34. What tendon has an intra-articular (instrasynovial) location in the knee joint?

1- Patellar
2- Popliteal
3- Semitendinosus

Dr. Ahmed Altaei


American Academy of Orthopaedic Surgeons
2002 Anatomy Self-Assessment Examination
Page 19
4- Semimembranosus
5- Biceps femoris

PREFERRED RESPONSE: 2

DISCUSSION: The popliteal tendon arises from the posterior aspect of the tibia and courses
through the knee joint through the popliteus hiatus of the lateral meniscus before attaching on the
lateral femur anterior to the lateral collateral ligament. It is the only tendon in the knee joint that
can be viewed directly on arthroscopy.

Kimura M, Shirakura K, Hasegawa A, Kobayashi Y, Udagawa E: Anatomy and pathophysiology of the popliteal tendon
area in the lateral meniscus: 1. Arthroscopic and anatomical investigation. Arthroscopy 1992;8:419-423.
Arnoczky SP, Skyhar MJ, Wickiewicz TL: Basic science of the knee, in McGinty JB (ed): Operative Arthroscopy. New
York, NY, Raven Press, 1991, pp 155-182.

35. A patient undergoes hip arthroscopy, and the pathology is seen in Figure 18. What
is the most likely diagnosis?

1- Femoral head articular defect


2- Acetabular articular defect
3- Haversian fat pad hypertrophy
4- Labral tear
5- Loose body

PREFERRED RESPONSE: 4

DISCUSSION: The motorized shaver is adjacent to the acetabular labrum, which is torn. The
femoral head and acetabulum are normal in appearance. Neither the fat pad nor a loose body is
identified.

Byrd JWT: Operative Hip Arthroscopy. New York, NY, Thieme Medical Publishers, 1998, pp 93-104.

36. Figure 19 shows an arthroscopic view from the


anterior lateral portal of the knee looking into
the suprapatella pouch. The use of an
electrothermal device during this procedure
most commonly causes significant postoperative
complications by damaging which of the
following structures?

1- Lateral facet articular cartilage of the patella


2- Peroneal nerve

Dr. Ahmed Altaei


American Academy of Orthopaedic Surgeons
2002 Anatomy Self-Assessment Examination
Page 20
3- Superior lateral geniculate artery
4- Inferior lateral geniculate artery
5- Lateral collateral ligament femoral insertion

PREFERRED RESPONSE: 3

DISCUSSION: While it is possible to damage any of these structures, unrecognized


intraoperative laceration without adequate coagulation of the superior lateral geniculate artery is
common. This can result in significant postoperative hemarthrosis and a return to surgery when
bleeding cannot be controlled.

Cash JD, Hughston JC: Treatment of acute patella dislocation. Am J Sports Med 1988;16:244-249.
Henry R, Goletz B, Williamson C: Lateral release in patello-femoral subluxation. Am J Sports Med 1986;14:121.

37. Figure 20 shows the resting and stress radiographs of a patient who has had pain
and feelings of instability after undergoing a total knee arthroplasty 1 year ago.
Which of the following ligaments is not functional and is therefore responsible for
the patients symptoms?

1- Anterior cruciate
2- Posterior cruciate
3- Medial collateral
4- Lateral collateral
5- Patellar tendon/ligament

PREFERRED RESPONSE: 2

DISCUSSION: The radiographs show posterior instability caused by an absent posterior cruciate
ligament. The tibia is significantly displaced posteriorly with respect to the femur. This can be
demonstrated with a lateral radiograph obtained with the knee in flexion. The anterior cruciate
ligament has been resected but is not responsible for the instability shown.

Incavo SJ, Churchill DL: The role of the posterior cruciate ligament in total knee arthroplasty. Techniques Orthop
1999;14:267-273.

38. On MRI, a nonsanguinous effusion has what appearance?

1- White (high-signal intensity) on T1- and T2-weighted images


2- White on T1-weighted images and black (low-signal intensity) on T2-weighted
images
3- Black on T1- and T2-weighted images
4- Black on T1-weighted images and white on T2-weighted images

Dr. Ahmed Altaei


American Academy of Orthopaedic Surgeons
2002 Anatomy Self-Assessment Examination
Page 21
5- Gray (intermediate-signal intensity) on T1-weighted images and white on T2-
weighted images

PREFERRED RESPONSE: 4

DISCUSSION: Nonbloody effusions that are greater than 1 mL are readily detected by MRI.
They appear black on T1-weighted images and white on T2-weighted images. A sanguinous
effusion is seen as white on T1-weighted images and black on T2-weighted images.

Resnick D, Kang HS: Synovial joints, in Resnick D, Kang HS (eds): Internal Derangements of Joints: Emphasis on MR
Imaging. Philadelphia, PA, WB Saunders, 1997, pp 49-53.
White EM: Magnetic resonance imaging in synovial disorders and arthropathy of the knee. Magn Reson Imaging Clin N
Am 1994;2:451-461.

39. Which of the following illustrations shown in Figures 21a through 21e correctly
shows the projection of the sacroiliac joint on the outer table of the ilium?

e d c

b a
1- a
2- b
3- c
4- d
5- e

PREFERRED RESPONSE: 3

Dr. Ahmed Altaei


American Academy of Orthopaedic Surgeons
2002 Anatomy Self-Assessment Examination
Page 22
DISCUSSION: The projection of the sacroiliac joint on the outer surface of the ilium should be
well understood to avoid violation of the joint during bone graft harvesting and to help in
insertion of the screw across the joint. The sacroiliac joint has superior and inferior limbs. The
average lengths of the superior and inferior limbs are 4.4 cm and 5.6 cm, respectively. The
average width of each limb is 2.0 cm. The average distance from the longitudinal axis of the
superior limb to the posterior superior iliac spine is 5.5 cm. The average longitudinal axis of the
inferior limb is 1.2 cm superior to the inferior margin of the posterior inferior iliac spine. The
average angle between the two axes is 93 degrees. Figure 21c most closely shows the projection
of the sacroiliac joint on the outer table of the ilium.

Waldrop JT, Ebraheim NA, Yeasting RA, Jackson WT: The location of the sacroiliac joint on the outer table of the posterior
ilium. J Orthop Trauma 1993;7:510-513.
Xu R, Ebraheim NA, Yeasting RA, Jackson WT: Anatomic considerations for posterior iliac bone harvesting. Spine
1996;21:1017-1020.

40. In children between the ages of 4 and 8 years, the major blood supply to the femoral
head comes from the

1- posterosuperior and posteroinferior branches of the medial femoral circumflex


artery.
2- anterior branch of the lateral femoral circumflex artery.
3- anterior branch of the medial femoral circumflex artery.
4- ascending branch of the lateral femoral circumflex artery.
5- artery of the ligamentum teres.

PREFERRED RESPONSE: 1

DISCUSSION: From birth until the age of 4 years, the primary blood supply to the femoral head
is from the medial and lateral circumflex arteries that traverse the femoral neck. After the age of
4 years, the contribution of the lateral femoral circumflex artery, which traverses the anterior
portion of the femoral neck, becomes negligible. The posterosuperior and posteroinferior
retinacular vessels, branches of the medial femoral circumflex artery, become the primary blood
supply to the epiphysis. The contribution of the artery of the ligamentum teres is minimal after
the age of 4 years.

Hughes LO, Beaty JH: Fractures of the head and neck of the femur in children. J Bone Joint Surg Am 1994;76:283-292.
Ogden JA: Changing patterns of proximal femoral vascularity. J Bone Joint Surg Am 1974;56:941-950.

41. What is the most anatomic location for placement of the femoral tunnel in anterior
cruciate ligament reconstruction?

1- As far superior in the notch as possible


2- As far posterior as possible on the lateral femoral condyle
3- As far posterior as possible on the medial femoral condyle
4- Directly across from the posterior cruciate femoral insertion

Dr. Ahmed Altaei


American Academy of Orthopaedic Surgeons
2002 Anatomy Self-Assessment Examination
Page 23
5- At residents ridge

PREFERRED RESPONSE: 2

DISCUSSION: It is critical for graft isometry and knee stability that the femoral tunnel be placed
as far posterior as possible on the lateral femoral condyle. Superiorly, the graft should be at the
one oclock position on the left knee. Residents ridge is a false posterior shelf that often seems
like the extreme posterior cortex. Abnormal tunnel placement results in a variety of
complications, including an unstable knee, early graft failure, and joint stiffness.

Johnson RJ, Beynnon BD, Nichols CE, Renstrom PA: The treatment of injuries of the anterior cruciate ligament. J Bone
Joint Surg Am 1992;74:140-151.
Beaty JH (ed): Orthopaedic Knowledge Update 6. Rosemont, IL, American Academy of Orthopaedic Surgeons, 1999, pp
533-557.

42. In the anterior approach (Smith-Petersen) to the hip, dissection is carried out
between muscles innervated by the

1- superior gluteal nerve laterally and the obturator nerve medially.


2- superior gluteal nerve laterally and the femoral nerve medially.
3- superior gluteal nerve medially and the femoral nerve laterally.
4- superior gluteal nerve medially and the inferior gluteal nerve laterally.
5- femoral nerve laterally and the obturator nerve medially.

PREFERRED RESPONSE: 2

DISCUSSION: In the Smith-Petersen approach to the hip, dissection is carried out between the
tensor fascia lata laterally (supplied by the superior gluteal nerve) and the sartorius and rectus
femoris medially (both supplied by the femoral nerve).

Crenshaw AH Jr: Surgical techniques and approaches, in Canale ST (ed): Campbells Operative Orthopaedics, ed 9. St
Louis, MO, Mosby-Year Book, 1998, vol 1, pp 82-85.
Williams PL, Warwick R, Dyson M, Bannister LH: Myology, in Grays Anatomy, ed 37. Edinburgh, Scotland, Churchill
Livingstone, 1989, pp 638-639.

43. What structure is marked Q in


the diagram of the brachial
plexus shown in Figure 22?

1- Medial cord
2- Lateral cord
3- Posterior cord
4- Upper trunk
5- Lower trunk

Dr. Ahmed Altaei


American Academy of Orthopaedic Surgeons
2002 Anatomy Self-Assessment Examination
Page 24
PREFERRED RESPONSE: 3

DISCUSSION: From proximal to distal, the brachial plexus is divided into roots, trunks,
divisions, and cords before forming specific peripheral nerve branches. The structure marked Q
is called the posterior cord because it lies posterior to the axially artery at the level of the cords.
Its terminal branches are the upper subscapular (V), thoracodorsal (W), lower subscapular (X),
axillary (F), and radial (G) nerves.

Anderson JE (ed): Grants Atlas of Anatomy, ed 7. Baltimore, MD, Williams and Wilkins, 1978, pp 6-24.
Last RJ: Anatomy: Regional and Applied, ed 6. London, England, Churchill Livingstone, 1978, p 63.

44. A sagittal T1-weighted MRI scan of the knee joint is shown in Figure 23. What
structure is identified by the arrow?

1- Popliteal tendon
2- Ligament of Humphrey
3- Anterior cruciate ligament
4- Posterior cruciate ligament
5- Lateral gastrocnemius tendon

PREFERRED RESPONSE: 4

DISCUSSION: On T1-weighted images, the posterior cruciate ligament is a low-signal (black)


structure that courses from the posterior aspect of the tibia to the medial femoral condyle. The
posterior cruciate ligament can appear as arcuate, U-shaped, or kinked. The other structures
have similar signal but different anatomic locations.

Gross ML, Grover JS, Bassett LW, Seeger LL, Finerman GA: Magnetic resonance imaging of the posterior cruciate
ligament: Clinical use to improve diagnostic accuracy. Am J Sports Med 1992;20:732-737.
Sonin AH, Fitzgerald SW, Friedman H, Hoff FL, Hendrix RW, Rogers LF: Posterior cruciate ligament injury: MR imaging
diagnosis and patterns of injury. Radiology 1994;190:455-458.

45. Figure 24 shows the arthroscopic view of


a patient with ankle impingement
syndrome. This is commonly seen after
high ankle sprains and represents
fibrotic granulation thickening of what
structure?

1- Talar dome fragment


2- Deltoid ligament

Dr. Ahmed Altaei


American Academy of Orthopaedic Surgeons
2002 Anatomy Self-Assessment Examination
Page 25
3- Anterior inferior tibiofibular ligament
4- Os trigonum
5- Spring ligament

PREFERRED RESPONSE: 3

DISCUSSION: Chronic anterior inferior tibiofibular ligament sprains can lead to thickening and
synovitis that catches or impinges dorsiflexion; patients often note painful clicking with
dorsiflexion eversion. The other structures are not affected by this injury.

Hopkinson WJ, St Pierre P, Ryan JB, Wheeler JH: Syndesmosis sprains of the ankle. Foot Ankle 1990;10:325-330.
Amendola A: Controversies in diagnosis and management of syndesmosis injuries of the ankle. Foot Ankle 1992;13:44-50.
Baxter DE: The Foot and Ankle in Sports. St Louis, MO, Mosby-Year Book, 1995, p 30.
Pfeffer GB (ed): Chronic Ankle Pain in the Athlete. Rosemont, IL, American Academy of Orthopaedic Surgeons, 2000, p
11.

46. The modified Brostrom lateral ankle ligamentous reconstruction uses which of the
following structures to provide supplementary stabilization?

1- One half of the peroneus brevis tendon


2- The entire peroneus brevis tendon
3- Peroneus longus tendon
4- Plantaris
5- Extensor retinaculum

PREFERRED RESPONSE: 5

DISCUSSION: The modified Brostrom lateral ankle ligament stabilization procedure uses the
remnants of the anterior talofibular and the calcaneofibular ligaments, supplemented by the
inferior retinaculum and the transferred talocalcaneal ligament to stabilize the lateral ankle.
Chrisman and associates described the use of one half of the peroneus brevis. Watson-Jones and
Evans used the entire peroneus brevis. The peroneus longus has been taken by mistake. The
plantaris has been used in triligamentous reconstruction.

Gould N, Seligson D, Gassman J: Early and late repair of lateral ligament of the ankle. Foot Ankle 1980;1:84-89.
Hamilton WG, Thompson FM, Snow SW: The modified Brostrom procedure for lateral ankle instability. Foot Ankle
1993;14:1-7.
Chrisman OD, Snook GA: Reconstruction of lateral ligament tears of the ankle: An experimental study and clinical
evaluation of seven patients treated by a new modification of the Elmslie procedure. J Bone Joint Surg Am 1969;51:904-
912.
Evans DL: Recurrent instability of the ankle: My method of surgical treatment. Proc R Soc Med 1953;46:343.
Watson-Jones R: Fractures and Joint Injuries, ed 3. Baltimore, MD, Williams and Wilkins, 1946, p 234.
Liu SH, Baker CL: Comparison of lateral ankle ligamentous reconstruction procedures. Am J Sports Med 1994;22:313-317.
Brostrom L: Sprained ankles: VI. Surgical treatment of chronic ligament ruptures. Acta Chir Scand 1966;132:551-565.

47. Figure 25 shows an arthroscopic thermal capsular shrinkage device being used in
the anterior inferior quadrant of a patient with a subluxating shoulder. Which of

Dr. Ahmed Altaei


American Academy of Orthopaedic Surgeons
2002 Anatomy Self-Assessment Examination
Page 26
the following neurologic complications is most frequently reported with this
technique?

1- Axillary nerve dysesthesia


2- Axillary nerve motor partial
paralysis
3- Suprascapular nerve neurapraxia
4- Musculocutaneous nerve
neurapraxia
5- Radial nerve sensory dysesthesia

PREFERRED RESPONSE: 1

DISCUSSION: The axillary nerve lies within millimeters of the anterior inferior capsule. The
inferior capsule is of varying thickness, and thermal energy used in shortening the ligament can
cause damage to the sensory fibers of the axillary nerve. Clinically, this is manifested as a burnt
skin sensation in the axillary nerve distribution area. The motor branch of the axillary nerve is
usually spared. The suprascapular nerve and the radial nerve are far from the shrinkage zone.
The musculocutaneous nerve, frequently at risk with open procedures, lies well anterior.

Fanton GS: Arthroscopic electrothermal surgery of the shoulder. Op Tech Sports Med 1998;6:157-160.
David TS, Drez DJ Jr: Electrothermally-assisted capsular shift. IEEE Eng Med Biol Mag 1998;17:102-104.

48. A coronal MRI scan through the shoulder joint is shown in Figure 26. The cyst
indicated by the arrow will most likely cause compression of what nerve?

1- Subscapular
2- Suprascapular
3- Axillary
4- Musculocutaneous
5- Medial pectoral

PREFERRED RESPONSE: 2

DISCUSSION: The MRI scan shows a ganglion cyst in the region of the spinoglenoid notch.
These are difficult to diagnose clinically but are readily apparent on MRI. They usually cause
compression of the suprascapular nerve and weakness of the infraspinatus and supraspinatus
muscles.

Dr. Ahmed Altaei


American Academy of Orthopaedic Surgeons
2002 Anatomy Self-Assessment Examination
Page 27
Resnick D, Kang HS (eds): Internal Derangements of Joints: Emphasis on MR Imaging. Philadelphia, PA, WB Saunders,
1997, pp 306-309.
Iannotti JP, Ramsey ML: Arthroscopic decompression of a ganglion cyst causing suprascapular nerve compression.
Arthroscopy 1996;12:739-745.

49. Which of the following ligaments is most commonly involved in posterolateral


rotatory instability of the elbow?

1- Annular
2- Lateral ulnar collateral
3- Anterior band of the medial collateral
4- Radial part of the lateral collateral
5- Posterior capsular

PREFERRED RESPONSE: 2

DISCUSSION: Recurrent posterolateral rotatory instability of the elbow is difficult to diagnose.


Such instability can be demonstrated only by the lateral pivot-shift test. The cause for this
condition is laxity of the ulnar part of the lateral collateral ligament, which allows a transient
rotatory subluxation of the ulnohumeral joint and a secondary dislocation of the radiohumeral
joint. The annular ligament remains intact, so the radioulnar joint does not dislocate. Treatment
consists of surgical reconstruction of the lax ulnar part of the lateral collateral ligament. The
anterior band is the most important part of the medial collateral which is lax in valgus instability
of the elbow.

Morrey BF: Acute and chronic instability of the elbow. J Am Acad Orthop Surg 1996;4:117-128.
ODriscoll SW, Bell DF, Morrey BF: Posterolateral rotatory instability of the elbow. J Bone Joint Surg Am 1991;73:440-
446.

50. Which of the following anatomic structures is labeled 6


in Figure 27?

1- A2 pulley
2- Graysons ligament
3- Clelands ligament
4- Triangular ligament
5- Sagittal band

PREFERRED RESPONSE: 1

DISCUSSION: The line labeled 6 points to the A2 pulley. This structure is the condensation of
the digital flexor tendon sheath corresponding to the proximal aspect of the proximal phalanx.
Graysons ligament is volar to the digital nerve and artery. Clelands ligament is dorsal to the
digital nerve and artery. The sagittal band anchors the extensor tendons over the

Dr. Ahmed Altaei


American Academy of Orthopaedic Surgeons
2002 Anatomy Self-Assessment Examination
Page 28
metacarpophalangeal joints. The triangular ligament connects the lateral bands just proximal to
the terminal tendon inserting onto the base of the distal phalanx.

Hollinshead WH: Anatomy for Surgeons: The Back and Limbs, ed 3. Philadelphia, PA, Harper and Row, 1982, p 467.
Tubiana R, McCullough CJ, Masquelet AC: An Atlas of Surgical Exposures of the Upper Extremity. Philadelphia, PA, JB
Lippincott, 1990, p 309.

51. A 41-year-old man who plays golf regularly has had ulnar-sided wrist pain for the
past several days after striking a tree root with a golf club. Examination reveals
significant pain with resisted flexion of the ring and small fingers and tenderness
over the hook of the hamate. Which of the following radiographic views would be
most helpful in identifying the pathology of this injury?

1- PA and lateral views of the wrist


2- PA, lateral, and oblique views of the hand
3- Scaphoid view
4- Bruertons view
5- Carpal tunnel view

PREFERRED RESPONSE: 5

DISCUSSION: The history and examination findings suggest an acute fracture of the hook of the
hamate. The radiographic study considered most helpful in identifying this type of fracture is the
carpal tunnel view. PA and lateral views of the wrist will not adequately visualize the hook of
the hamate. Bruertons view is intended for the assessment of the metacarpophalangeal joints.
Pathology would not be suspected in the scaphoid, metacarpals, or the phalanges, so the scaphoid
view and the PA, lateral, and oblique views of the hand would not be helpful.

Green DP, Hotchkiss RN, Pederson WC (eds): Operative Hand Surgery, ed 4. New York, NY, Churchill Livingstone, 1999,
p 855.
Manske PR (ed): Hand Surgery Update. Rosemont, IL, American Society for Surgery of the Hand, 1994, pp 77-84.

52. What is the primary limiting membrane and mechanical support for the periphery
of the physis?

1- Perichondrial ring of La Croix


2- Groove of Ranvier
3- Zone of provisional calcification
4- Last intact transverse septum
5- Primary spongiosa

PREFERRED RESPONSE: 1

DISCUSSION: The perichondrial fibrous ring of La Croix acts as a limiting membrane that
provides mechanical support for the bone-cartilage junction of the growth plate. It is continuous
with the ossification groove of Ranvier, which contributes chondrocytes for the increase in width
of the growth plate. The zone of provisional calcification lies at the bottom of the hypertrophic
Dr. Ahmed Altaei
American Academy of Orthopaedic Surgeons
2002 Anatomy Self-Assessment Examination
Page 29
zone and is the site of initial calcification of the matrix. It is quite weak and usually is the
cleavage plane for fractures; therefore, it does not qualify as mechanical support. The last intact
transverse septum separates the zone of provisional calcification from the primary spongiosa and
provides no real support to the physis. The primary spongiosa is the part of the metaphysis
nearest the physis.

Netter FH: Growth plate, in Woodburne RT, Crelin ES, Kaplan FS, Dingle RV (eds): The Ciba Collection of Medical
Illustrations. Summit, NJ, Ciba-Geigy Corporation, 1987, vol 8, pp 166-167.
Asher MA (ed): Orthopaedic Knowledge Update 1. Chicago, IL, American Academy of Orthopaedic Surgeons, 1984, pp
15-28.

53. What normal tissue has a low signal intensity (appears black) on both T1- and T2-
weighted images?

1- Tendon
2- Fat
3- Joint fluid
4- Intervertebral disk
5- Bone marrow

PREFERRED RESPONSE: 1

DISCUSSION: Tendons, cortical bone, ligaments, menisci, and fibrous tissue will show low
signal intensity (SI) on both T1- and T2-weighted images. Fat-containing tissues, such as
subcutaneous fat and bone marrow, will show high SI on T1-weighted images and low SI on T2-
weighted images. Tissues with high water content, such as joint fluid, intervertebral disk, and
edema, will show low SI on T1-weighted images and high SI on T2-weighted images.

Kasser JR (ed): Orthopaedic Knowledge Update 5. Rosemont, IL, American Academy of Orthopaedic Surgeons, 1996, pp
65-70.
Slucky AV, Potter HG: Use of magnetic resonance imaging in spinal trauma: Indications, techniques, and utility. J Am
Acad Orthop Surg 1998;6:134-145.

54. A positive Froment sign indicates weakness of which of the following muscles?

1- First dorsal interosseous


2- Adductor pollicis
3- Opponens pollicis
4- Flexor pollicis brevis
5- Abductor pollicis longus

PREFERRED RESPONSE: 2

DISCUSSION: Thumb adduction is powered by the adductor pollicis (ulnar nerve). Testing
involves having the patient forcibly hold a piece of paper between the thumb and radial side of

Dr. Ahmed Altaei


American Academy of Orthopaedic Surgeons
2002 Anatomy Self-Assessment Examination
Page 30
the index proximal phalanx. When this muscle is weak or nonfunctioning, the thumb
interphalangeal joint flexes with this maneuver, resulting in a positive Froment sign. The paper
is held by action of the thumb flexion (flexor pollicis longus and flexor pollicis brevis; median
innervated).

Burton RI: The Hand: Examination and Diagnosis. Chicago, IL, American Society for Surgery of the Hand, 1978, pp 26-27.

55. Figure 28 shows an AP radiograph of a 54-year-old woman who underwent lumbar


laminectomy and fusion at the L4 and L5 levels with placement of a bone stimulator
8 years ago. She also underwent a left total hip arthroplasty 2 years ago; aspiration
of that joint now reveals that it is infected with a gram-positive cocci organism.
History is also significant for IV drug use and human immunodeficiency virus
(HIV). The patient reports fever, chills, and left flank and abdominal pain.
Examination reveals significant pain with resisted left hip flexion and passive hip
extension. She also has lumbar hyperlordosis. Which of the following studies would
best identify the underlying cause of her infection?

1- Abdominal and pelvic ultrasound


2- CT of the abdomen and pelvis
3- Technetium Tc 99m three-phase bone
scan
4- Abdominal radiographic series
5- Gallium-67 citrate scan

PREFERRED RESPONSE: 2

DISCUSSION: The patients clinical signs (fever and flank, hip, and abdominal pain) suggest a
primary iliopsoas abscess. With an increased patient population who abuse drugs and/or who are
HIV-positive, iliopsoas abscess may be more prevalent because of systemic bacterial seeding and
may be potentially unrecognized. Diagnostic imaging studies provide a better understanding of
the anatomic magnitude of the infection, give concrete confirmation of the diagnosis, and may
suggest an underlying cause. Neither standard abdominal radiographs nor ultrasound studies are
sensitive enough to be diagnostic of this disease process. CT has been established as the
standard study for identifying the underlying cause of this abscess. The hip infection has most
likely developed as a result of hematogenous spread from an infected skin lesion from the
patients IV drug use.

Dr. Ahmed Altaei


American Academy of Orthopaedic Surgeons
2002 Anatomy Self-Assessment Examination
Page 31
Santaella RO, Fishman EK, Lipsett PA: Primary vs secondary iliopsoas abscess: Presentation, microbiology, and treatment.
Arch Surg 1995;130:1309-1313.
Malhotra R, Singh KD, Bhan S, Dave PK: Primary pyogenic abscess of the psoas muscle. J Bone Joint Surg Am
1992;74:278-284.

56. A 55-year-old woman who underwent a left total hip arthroplasty 8 months ago
using a modified Hardinger approach reports a persistent painless limp.
Examination reveals that when she is not using a cane, she lurches to the left during
weight bearing on the left lower extremity. An AP radiograph is shown in Figure
29. Which of the following hip muscle groups should be strengthened to improve
the gait abnormality?

1- External rotators
2- Extensors
3- Abductors
4- Flexors
5- Adductors

PREFERRED RESPONSE: 3

DISCUSSION: The modified Hardinger approach includes a partial anterior trochanteric


osteotomy creating a trochanteric wafer (as seen on the radiograph) that is displaced anterior and
medial in continuity with the gluteus medius and vastus lateralis. Failure of abductor
reattachment, migration of the trochanter, nonunion of the osteotomy site, and excessive splitting
of the gluteus medius muscle causing injury to the inferior branch of the superior gluteal nerve
can result in weakness of the abductor mechanism. Abductor strength should be evaluated with
the patient lying on the opposite side and elevating the affected limb. Although slight weakness
may manifest itself as a limp only after prolonged muscular activity, significant weakness results
in a constant limp without associated discomfort.

Morrey BF (ed): Joint Replacement Arthroplasty. New York, NY, Churchill Livingstone, 1991, pp 512-526.
Kasser JR (ed): Orthopedic Knowledge Update 5. Rosemont, IL, American Academy of Orthopaedic Surgeons, 1996, pp
389-426.

57. Figure 30 shows an axial cross


section of extensor tendon anatomy
in zone 7 of the wrist. What letter
best depicts the location of the
posterior interosseous nerve?

Dr. Ahmed Altaei


American Academy of Orthopaedic Surgeons
2002 Anatomy Self-Assessment Examination
Page 32
1- A
2- B
3- C
4- D
5- E

PREFERRED RESPONSE: 3

DISCUSSION: The posterior interosseous nerve in contained in the floor of the fourth dorsal
compartment of the wrist, which is labelled C in this diagram.

Tubiana R, McCullough CJ, Masquelet AC: An Atlas of Surgical Exposures of the Upper Extremity. Philadelphia, PA, JB
Lippincott, 1990, p 224.
Hollinshead WH: Anatomy for Surgeons: The Back and Limbs, ed 3. Philadelphia, PA, Harper and Row, 1982, p 430.

58. Figures 31a and 31b show the T1- and T2-weighted MRI scans of a patients knee
joint. What is the most likely diagnosis?

1- Torn anterior cruciate


ligament
2- Torn medial meniscus
3- Staphylococcus
infection
4- Rheumatoid arthritis
5- Tibial plateau fracture

PREFERRED RESPONSE: 5

DISCUSSION: The scans show a lipohemarthrosis. There is the characteristic layering of a


superior zone containing fat (high signal intensity), a central zone containing serum (low signal
intensity), and an inferior zone that contains red blood cells (low signal intensity). The most
common cause of a lipohemarthrosis is an intra-articular fracture with leakage of marrow fat into
the joint.

Resnick D, Kang HS: Synovial joints, in Resnick D, Kang HS (eds): Internal Derangements of Joints: Emphasis on MR
Imaging. Philadelphia, PA, WB Saunders, 1997, pp 49-53.
Kier R, McCarthy SM: Lipohemarthrosis of the knee: MR imaging. J Comput Assist Tomogr 1990;14:395-396.

Dr. Ahmed Altaei


American Academy of Orthopaedic Surgeons
2002 Anatomy Self-Assessment Examination
Page 33
59. A 15-year-old boy reports feeling a pop and notes sudden giving way of the left knee
while playing basketball. He has immediate pain and swelling in the knee. An AP
radiograph is shown in Figure 32. A small avulsion fragment from the lateral tibial
margin is the only finding. What is the most likely diagnosis?

1- Avulsion of the lateral collateral ligament


2- Avulsion of the pes anserinus
3- Avulsion of the iliotibial band
4- Tear of the anterior cruciate ligament
5- Tear of the posterior cruciate ligament

PREFERRED RESPONSE: 4

DISCUSSION: An avulsion fracture from the lateral tibial margin carries the eponym Segond
fracture and is pathognomonic for an anterior cruciate ligament (ACL) tear. The fragment is
located posterior to Gerdys tubercle and is superior and anterior to the fibular head. It
represents an avulsion of the lateral capsular ligament of the knee and is caused by the same
mechanism that causes the ACL tear. The pes anserinus is the insertion point of the medial
hamstrings and would not be affected in a lateral avulsion injury. The posterior cruciate
ligament may be seen on a lateral view if associated with an avulsion fragment, but a tear of the
PCL generally cannot be diagnosed on an AP view. The insertion of the iliotibial band is broad
and is unlikely to produce an avulsion injury such as that seen in the radiograph. This view is
not consistent with the appearance of a lateral collateral ligament injury.

Beaty JH (ed): Orthopaedic Knowledge Update 6. Rosemont, IL, American Academy of Orthopaedic Surgeons, 1999, pp
533-557.
Larson RL, Tailon M: Anterior cruciate ligament insufficiency: Principles of treatment. J Am Acad Orthop Surg
1994;2:26-35.

60. What neurovascular structure is most at risk when performing an inside-out repair
of the posterior horn of the medial meniscus?

1- Popliteal artery
2- Peroneal nerve
3- Saphenous nerve
4- Tibial nerve
5- Sciatic nerve

PREFERRED RESPONSE: 3

Dr. Ahmed Altaei


American Academy of Orthopaedic Surgeons
2002 Anatomy Self-Assessment Examination
Page 34

DISCUSSION: The saphenous nerve is located on the posterior medial aspect of the knee and
must be protected when performing an inside-out repair of the medial meniscus. The peroneal
nerve is most at risk with lateral meniscal repairs. The other structures usually are not at risk
with meniscal repair.

Cannon WD Jr, Morgan CD: Meniscal repair: Arthroscopic repair techniques. Instr Course Lect 1994;43:77-96.
Scott GA, Jolly BL, Henning CE: Combined posterior incision and arthroscopic intra-articular repair of the meniscus: An
examination of factors affecting healing. J Bone Joint Surg Am 1986;68:847-861.

61. Figure 33 shows the AP and lateral radiographs of an obese 58-year-old man who
underwent a cementless total hip arthroplasty 6 years ago. He reports no pain, and
examination reveals a normal gait and painless hip range of motion. What is the
most likely diagnosis?

1- Osteolysis because of
polyethylene debris
2- Spot weld
3- Osteosclerotic pedestal
4- Cement fragmentation
5- Allergic reaction to titanium

PREFERRED RESPONSE: 1

DISCUSSION: Osteolysis of an otherwise well-functioning total hip arthroplasty is a recognized


complication, and its radiographic appearance is typical, as shown here. Distal osteolysis, such
as that shown here, is more prevalent when there is noncircumferential sealing of the proximal
femoral canal.

Pellicci PM, Tria AJ Jr, Garvin KL (eds): Orthopaedic Knowledge Update: Hip and Knee Reconstruction 2. Rosemont, IL,
American Academy of Orthopaedic Surgeons, 2000, pp 175-180.
Engh CA, Massin P, Suthers KF: Roentgenographic assessment of the biologic fixation of porous-surfaced femoral
components. Clin Orthop 1990;257:107-128.

62. Which of the following muscles attaches to the coracoid process of the scapula?

1- Subscapularis
2- Supraspinatus
3- Pectoralis minor
4- Long head of the biceps brachii
5- Serratus anterior

PREFERRED RESPONSE: 3

Dr. Ahmed Altaei


American Academy of Orthopaedic Surgeons
2002 Anatomy Self-Assessment Examination
Page 35
DISCUSSION: The insertion of the pectoralis minor is on the base of the coracoid process. The
coracoid helps define the interval between the subscapularis and supraspinatus muscles but
neither attaches to it. The coracobrachialis and short head of biceps attach to the tip of the
coracoid but are not listed as options. The long head of the biceps attaches to the supraglenoid
tubercle. The serratus arises from the vertebral border of the scapula.

Jobe CM: Anatomy and surgical approaches, in Jobe FW (ed): Operative Techniques in Upper Extremity Sports Injuries. St
Louis, MO, Mosby, 1996, pp 140-142.
Jobe CM: Gross anatomy of the shoulder, in Rockwood CA, Matsen FA III (eds): The Shoulder. Philadelphia, PA, WB
Saunders, 1990, pp 49-66.

63. Turf toe typically involves injury to which of the following structures of the great
toe?

1- Nail bed
2- Extensor tendon
3- Flexor tendon
4- Capsule of the first metatarsophalangeal joint
5- Distal phalanx of the first toe

PREFERRED RESPONSE: 4

DISCUSSION: The term turf toe includes a range of injuries of the capsuloligamentous complex
of the first metatarsophalangeal joint with or without osteochondral fracture of the first
metatarsal head or one of the sesamoids. The mechanism of injury is hyperextension.

Clanton TO, Butler JE, Eggert A: Injuries to the metatarsophalangeal joints in athletes. Foot Ankle 1986;7:162-176.
Sammarco GJ: How I manage turf toe. Phys Sports Med 1988;16:113-118.

64. Figures 34a through 34c show an axial proton density (spin echo long TR, short TE)
image, a sagittal inversion recovery (STIR) image, and a sagittal T1-weighted (short
TR, short TE) image of the left thigh. What is the most likely diagnosis?

Dr. Ahmed Altaei


American Academy of Orthopaedic Surgeons
2002 Anatomy Self-Assessment Examination
Page 36
1- Soft-tissue foreign body
2- Lipoma
3- Hematoma
4- Pyomyositis
5- Liposarcoma

PREFERRED RESPONSE: 3

DISCUSSION: The images reveal a region of increased signal within the rectus femoris muscle
with mild, ill-defined surrounding edema. The presence of high intensity signal on the T1-
weighted image favors acute blood, in this case associated with a rectus femoris muscle tear or
fatty tissue. However, because of fat suppression, a fatty lesion or lipoma would be dark on
STIR, rather than bright as in this image. Most foreign bodies are low intensity signal and if
small, are difficult to evaluate with MRI. The lack of adjacent subcutaneous soft-tissue edema or
surrounding fluid makes pyomyositis an unlikely diagnosis.

El-Khoury G: MRI of the Musculoskeletal System. Philadelphia, PA, JB Lippincott, 1998, p 123.

65. The artery located within the substance of the coracoacromial ligament is a branch
of what artery?

1- Thoracoacromial
2- Anterior circumflex humeral
3- Posterior circumflex humeral
4- Subscapular
5- Thyrocervical

PREFERRED RESPONSE: 1

DISCUSSION: The acromial branch of the thoracoacromial artery courses along the medial
aspect of the coracoacromial ligament and may be encountered when performing an open or
arthroscopic subacromial decompression. Bleeding can be controlled by ligation of its branch
from the thoracoacromial artery. The other arteries may be injured in other surgical exposures of
the shoulder.

Esch JC, Baker CL: The shoulder and elbow, in Whipple TL (ed): Arthroscopic Surgery. Philadelphia, PA, JB Lippincott,
1993, pp 65-66.
Woodburne RT (ed): Essentials of Human Anatomy, ed 2. New York, NY, Oxford University Press, 1983, pp 75-76.

66. Figures 35a and 35b show the axial T2-weighted and coronal T1-weighted MRI
scans of a patient who has enlargement of the right thigh. What is the most likely
diagnosis?

Dr. Ahmed Altaei


American Academy of Orthopaedic Surgeons
2002 Anatomy Self-Assessment Examination
Page 37

1- Conventional osteosarcoma
2- Periosteal osteosarcoma
3- Ewings sarcoma
4- Myositis ossificans
5- Subperiosteal chondroma

PREFERRED RESPONSE: 2

DISCUSSION: The images show a large, almost circumferential, mass surrounding the diaphysis
of the femur. The intramedullary signal is normal with minimal cortical destruction, both
findings that should be abnormal in conventional osteosarcoma and Ewings sarcoma. There are
very low-signal striations representing osteoid formation that would have a sunburst radiographic
pattern. This indicates an osteogenic lesion. Myositis ossificans is not indicated because studies
would reveal zonal ossification starting in the periphery rather than the more central pattern seen
in this patient. This appearance is typical for periosteal osteosarcoma.

El-Khoury G: MRI of the Musculoskeletal System. Philadelphia, PA, JB Lippincott, 1998, p 176 .

67. Figures 36a and 36b show the MRI scans of a patient who has shoulder weakness.
What is the most likely diagnosis?

Dr. Ahmed Altaei


American Academy of Orthopaedic Surgeons
2002 Anatomy Self-Assessment Examination
Page 38
1- Suprascapular nerve entrapment
2- Supraspinatus and infraspinatus tendon tear
3- Muscular dystrophy
4- Thoracic outlet syndrome
5- Spinal accessory nerve disruption

PREFERRED RESPONSE: 1

DISCUSSION: The sagittal image reveals increased signal and decreased size of the
supraspinatus and infraspinatus muscles, indicating muscle atrophy. The rotator cuff tendon
signal is normal. The subscapularis and teres minor muscles are unaffected. Muscular dystrophy
and thoracic outlet syndrome would be expected to have a more global effect. Although
muscular atrophy can occur in the setting of a rotator cuff tear, the coronal image shows an intact
supraspinatus. The suprascapular nerve supplies the supraspinatus and infraspinatus muscles.
Therefore, suprascapular nerve entrapment would result in atrophy of these muscles with sparing
of the surrounding musculature. Any lesion within the suprascapular notch, including neoplastic
disease, a venous varix, or neuroma, can place pressure on the suprascapular nerve.
Suprascapular nerve entrapment most commonly results from extension of a paralabral cyst or
ganglion, often with associated labral pathology. Spinal accessory nerve disruption would show
trapezius muscle atrophy.

Resnick D, Kang HS (eds): Internal Derangement of Joints: Emphasis on MR Imaging. Philadelphia, PA, WB Saunders,
1997, pp 308-317.
El-Khoury G: MRI of the Musculoskeletal System. Philadelphia, PA, JB Lippincott, 1998, p 123.

68. Figure 37 shows the T2-weighted MRI scan of the hip joint. What structure is
labeled A?

1- Ischiofemoral ligament
2- Ligamentum teres
3- Obturator internus
4- Piriformis
5- Coccygeus

PREFERRED RESPONSE: 3

DISCUSSION: The obturator internus originates on the obturator membrane and adjacent bone,
including the quadrilateral plate, and exits the lesser sciatic notch to insert on the posterior
medial greater trochanter. The structure labeled C is the pectineus, B is the sartorius, and D is
the gluteus medius.

Last RJ: Anatomy: Regional and Applied, ed 6. London, England, Churchill Livingstone, 1978, pp 145-150, 324.

Dr. Ahmed Altaei


American Academy of Orthopaedic Surgeons
2002 Anatomy Self-Assessment Examination
Page 39
Anderson JE (ed): Grants Atlas of Anatomy, ed 7. Baltimore, MD, Williams & Wilkins, 1978, plate 4-46.

69. The great medullary artery, also known as the Adamkiewicz artery, originates from
which of the following arteries?

1- Left posterior intercostal between T8 and T12


2- Posterior spinal between T1 and T12
3- Anterior spinal between T4 and T6
4- Superior phrenic
5- Inferior phrenic

PREFERRED RESPONSE: 1

DISCUSSION: The great medullary artery originates as a direct or indirect branch of the left
posterior intercostal artery, usually between T8 and T12. It becomes intradural and crosses over
one to three disk spaces before turning to the midline where it anastomoses with the anterior
spinal artery. Injury to this artery can result in devastating ischemia of the lower spinal cord.

Lu J, Ebraheim NA, Biyani A, Brown JA, Yeasting RA: Vulnerability of great medullary artery. Spine 1996;21:1852-1855.
Rogers FB, Osler TM, Shackford SR, Wald SL: Isolated stab wound to the artery of Adamkiewicz: Case report and review
of the literature. J Trauma 1997;43:549-551.

70. A patient who underwent total knee arthroplasty now reports a loss of sensation in
the area circled in Figure 38. This area is innervated by which of the following
nerves?

1- Infrapateller branch of the saphenous


2- Ascending branch of the peroneal
3- Anterior branch of the femoral
4- Posterior cutaneous of the thigh
5- Lateral cutaneous of the thigh

PREFERRED RESPONSE: 1

DISCUSSION: The saphenous nerve follows the saphenous vein, giving off the infrapatellar
branch that crosses the knee anteriorly to supply the peripatellar skin. A longitudinal incision
can interrupt the nerve, leaving the terminal distribution without sensation.

Last RJ: Anatomy: Regional and Applied, ed 6. London, England, Churchill Livingstone, 1978, pp 140-150.

Dr. Ahmed Altaei


American Academy of Orthopaedic Surgeons
2002 Anatomy Self-Assessment Examination
Page 40
71. During an anterior approach to the shoulder, excessive traction on the conjoined
tendon is most likely to result in loss of

1- elbow flexion.
2- shoulder flexion.
3- shoulder internal rotation.
4- shoulder abduction.
5- forearm pronation.

PREFERRED RESPONSE: 1

DISCUSSION: The musculocutaneous nerve travels through the conjoined tendon


approximately 8 cm distal to the tip of the acromion. The musculocutaneous nerve innervates
the biceps muscle and the bracialis muscle, both of which are responsible for elbow flexion.
Shoulder flexion is facilitated by the anterior fibers of the deltoid muscle (axillary nerve) and the
supraspinatus muscle (suprascapular nerve). The subscapular muscle facilitates internal rotation
of the shoulder (upper and lower subscapularis nerve). Shoulder abduction is performed by the
deltoid muscle (axillary nerve), and forearm pronation is facilitated by the pronator teres (median
nerve).

Hollinshead WH: Anatomy for Surgeons: The Back and Limbs, ed 3. Philadelphia, PA, Harper and Row, 1982, pp 391-393.
Hoppenfeld S, deBoer P: Surgical Exposures in Orthopaedics: The Anatomic Approach, ed 2. Philadelphia, PA, Lippincott-
Raven, 1992, pp 2-49.

72. Figure 39 shows the sagittal T1-weighted MRI scan of a 27-year-old man who
twisted his knee 2 weeks ago. The arrow is pointing to

1- a physeal scar.
2- a femoral stress fracture.
3- a normal growth plate.
4- Loosers line.
5- an abnormal growth
plate.

PREFERRED RESPONSE: 1

DISCUSSION: The arrow identifies a transverse dark line that represents primary trabeculae of
the physeal scar. A similar finding is seen in the proximal tibia. These lines may persist
indefinitely. They do not represent ongoing growth, an abnormally open physeal plate, a stress
fracture, or Loosers line (fatigue fracture in osteomalacia).
El-Khoury G: MRI of the Musculoskeletal System. Philadelphia, PA, JB Lippincott, 1998, p 123.

Dr. Ahmed Altaei


American Academy of Orthopaedic Surgeons
2002 Anatomy Self-Assessment Examination
Page 41
73. Figure 40 shows the AP radiograph of a 55-year-old man who reports left knee pain.
Which of the following conditions is least likely to produce this radiographic
presentation?

1- Hemochromatosis
2- Alkaptonuria
3- Wilsons disease
4- Septic arthritis
5- Calcium pyrophosphate dihydrate crystal
deposition

PREFERRED RESPONSE: 4

DISCUSSION: The radiograph reveals densities within the articular cartilage of the knee
commonly referred to as chondrocalcinosis. The term chondrocalcinosis refers to the presence of
calcium-containing crystals detected as radiodensities in cartilage. Calcium-containing crystals
other than calcium pyrophosphate dihydrate may also deposit in articular cartilage and menisci,
producing both radiographically detectable densities in cartilage and joint inflammation or
degeneration. Hemochromatosis, alkaptonuria (ochronosis), and Wilsons disease are
characterized by cellular deposition of iron, calcium, and copper ions, respectively, into various
tissues including articular cartilage and can give this appearance. Septic arthritis does not
usually cause chondrocalcinosis.

Klippel JH (ed): Primer on the Rheumatic Diseases, ed 11. Atlanta, GA, Arthritis Foundation, 1997, pp 226-229 and 328-
331.
Resnick D, Wayama G: Diagnosis of Bone and Joint Disorders, ed 2. Philadelphia, PA, WB Saunders, 1988, pp 1675, 1779.

74. Figure 41 shows the MRI scan of


a 39-year-old man who has severe
left groin and anterior thigh pain.
What is the most likely diagnosis?

1- Osteoarthritis
2- Rheumatoid arthritis
3- Synovial chondromatosis
4- Gout
5- Osteonecrosis

PREFERRED RESPONSE: 5

Dr. Ahmed Altaei


American Academy of Orthopaedic Surgeons
2002 Anatomy Self-Assessment Examination
Page 42
DISCUSSION: The MRI scan shows near complete involvement of the femoral head with bone
marrow changes and some collapse of the necrotic segment. This is most suggestive of
osteonecrosis.

Urbaniak JR, Jones JP Jr (eds): Osteonecrosis: Etiology, Diagnosis, and Treatment. Rosemont, IL, American Academy of
Orthopaedic Surgeons, 1997.

75. Iliosacral screws placed for stabilization of posterior pelvic ring injuries (eg,
sacroiliac dislocation) that exit the sacrum anteriorly are most likely to injure which
of the following structures?

1- L4 nerve root
2- L5 nerve root
3- S1 nerve root
4- Internal iliac artery
5- External iliac artery

PREFERRED RESPONSE: 2

DISCUSSION: Iliosacral screws have gained popularity for posterior stabilization of pelvic ring
disruptions, but complications attributed to incorrect placement are a clinical problem. The L5
nerve root is at greatest risk and is in closest proximity to a malpositioned screw (exiting the
sacrum). The L4 root is more anterior at this level. The S1 root is still intraosseous at this level
and is at risk but not from the screw exiting anteriorly at this level. The arteries are at risk but
are more anterior and are at less risk than the L5 nerve root.

Ebraheim NA, Haman SP, Xu R, Stanescu S, Yeasting RA: The lumbosacral nerves in relation to dorsal SI screw placement
and their locations on plain radiographs. Orthopedics 2000;23:245-247.

76. Figure 42 is a transverse MRI scan of the left shoulder. The arrow points to which
of the following structures?

1- Anterior labrum
2- Long head of the biceps tendon
3- Middle glenohumeral ligament
4- Lower glenohumeral ligament
5- Axillary nerve

PREFERRED RESPONSE: 2

DISCUSSION: The figure shows an axial image of the shoulder immediately inferior to the
coracoid process. The subscapularis tendon, which can be traced from the myotendinous

Dr. Ahmed Altaei


American Academy of Orthopaedic Surgeons
2002 Anatomy Self-Assessment Examination
Page 43
junction, is torn and detached from its lesser tuberosity attachment on the humerus. Lateral to
the lesser tuberosity, the bicipital groove is empty. The arrow points to the subluxated biceps
tendon. Superficial fibers of the subscapularis tendon are contiguous with the biceps
retinaculum, which covers the bicipital groove and hold the biceps tendon in place. The vast
majority of subscapularis tendon tears result in disruption of the biceps retinaculum with
resultant subluxation of the tendon.

Resnick D, Kang HS (eds): Internal Derangement of Joints: Emphasis on MR Imaging. Philadelphia, PA, WB Saunders,
1997, pp 308-317.
El-Khoury G: MRI of the Musculoskeletal System. Philadelphia, PA, JB Lippincott, 1998, p 234.

77. Within the menisci, the majority of the large collagen fiber bundles are oriented in
what configuration?

1- Radially
2- Circumferentially
3- Vertically
4- Obliquely
5- Randomly

PREFERRED RESPONSE: 2

DISCUSSION: The majority of large collagen fibers within the menisci are oriented
circumferentially. It is these fibers that develop the hoop stress with compressive loading of the
menisci. Most meniscal tears are longitudinal and occur between these circumferential fibers.

Mow VC, et al: Structure and function relationships of the menisci of the knee, in Mow VC, Arnoczky SP, Jackson DW
(eds): Knee Meniscus: Basic and Clinical Foundations. New York, NY, Raven Press, 1992, pp 37-57.
DeHaven KE, Arnoczky SP: Meniscus repair: Basic science, indications for repair, and open repair. Instr Course Lect
1994;43:65-76.

78. For halo traction, what is the preferred site for anterior pin placement?

1- Below the head equator, above the supraorbital ridge, and 3.5 cm lateral to the
midline
2- Below the head equator, above the supraorbital ridge, and 4.5 cm lateral to the
midline
3- Below the head equator, below the supraorbital ridge, and 4.5 cm lateral to the
midline
4- Above the head equator, above the supraorbital ridge, and 3.5 cm lateral to the
midline
5- Above the head equator, above the supraorbital ridge, and 4.5 cm lateral to the
midline

PREFERRED RESPONSE: 2

Dr. Ahmed Altaei


American Academy of Orthopaedic Surgeons
2002 Anatomy Self-Assessment Examination
Page 44

DISCUSSION: The safe zone for anterior halo pin insertion is marked laterally by the anterior
border of the temporalis muscle (to avoid penetration of this muscle and relative thin cortex of
the skull). Medially, the pin should be placed 4.5 cm lateral to the midline to avoid injury to the
supraorbital nerve or the frontal sinus. The safe area is marked superiorly by the head equator to
avoid cephalad migration of the pin and inferiorly by the supraorbital ridge to prevent
displacement or penetration into the orbit.

Ebraheim NA, Lu J, Biyani A, Brown JA: Anatomic considerations of halo pin placement. Am J Orthop 1996;25:754-756.

79. A 12-year-old boy has had progressive pain and flatfeet for the past year. Pain is
increased with weight-bearing activities. Examination reveals that subtalar motion
is absent. On standing, the patient has obvious hindfoot valgus and loss of the
normal arch bilaterally. Plain radiographs are shown in Figures 43a through 43c,
and a CT scan is shown in Figure 43d. What is the most likely diagnosis?

1- Peroneal spastic flatfoot


2- Flexible flatfoot with a short Achilles tendon
3- Calcaneonavicular coalition
4- Talocalcaneal coalition
5- Posterior tibial tendon dysfunction

PREFERRED RESPONSE: 4

Dr. Ahmed Altaei


American Academy of Orthopaedic Surgeons
2002 Anatomy Self-Assessment Examination
Page 45
DISCUSSION: The axial views show fusion of the talus and calcaneus at the medial facet
(talocalcaneal coalition). Peroneal spastic flatfoot is a descriptive term applying to the symptoms
of painful flatfoot associated with apparent peroneal spasm and is sometimes caused by tarsal
coalition; however, this is not the most appropriate diagnosis for this patient. Flexible flatfoot
with a short Achilles tendon often causes symptoms similar to the ones listed above, but subtalar
motion should be normal. A diagnosis of calcaneonavicular coalition can be made based on
plain oblique views of the foot but is not seen in these views. Posterior tibial tendon dysfunction
in the absence of other pathology is uncommon in children.

Vincent KA: Tarsal coalition and painful flatfoot. J Am Acad Orthop Surg 1998;6:274-281.
Beaty JH (ed): Orthopaedic Knowledge Update 6. Rosemont, IL, American Academy of Orthopaedic Surgeons, 1999, pp
583-595.

80. When performing ankle arthroscopy through the anterolateral portal, what
anatomic structure is at greatest risk?

1- Anterior tibialis tendon


2- Anterior tibial artery
3- Sural nerve
4- Deep peroneal nerve
5- Superficial peroneal nerve

PREFERRED RESPONSE: 5

DISCUSSION: The superficial branch of the peroneal nerve travels subcutaneously anterior to
the lateral malleolus at the ankle. It can be easily damaged by deep penetration of the knife
blade when making this portal or when passing shavers in and out of the portal. Anesthesia or
dysesthesia from laceration or neuroma formation can cause significant postoperative morbidity.
The anterior tibialis tendon, anterior tibial artery, and the deep peroneal nerve are located much
more anterior and central on the ankle. The sural nerve is posterior lateral to the ankle and is not
at risk from this portal.

Ferkel RD, Heath DD, Guhl JF: Neurological complications of ankle arthroscopy. Arthroscopy 1996;12:200-208.
Cooper PS, Murray TF Jr: Arthroscopy of the foot and ankle in the athlete. Clin Sports Med 1996;15:805-824.

81. Figure 44 shows the AP radiograph of the hip of a


patient who underwent screw fixation of the
acetabulum. Which of the following structures is at
least risk for injury during screw placement in the
acetabular component?
1- Common iliac artery
2- Superior gluteal artery
3- Obturator artery
4- Sciatic nerve
5- External iliac vein
Dr. Ahmed Altaei
American Academy of Orthopaedic Surgeons
2002 Anatomy Self-Assessment Examination
Page 46

PREFERRED RESPONSE: 1

DISCUSSION: Acetabular screws are inserted to supplement fixation. The acetabular


component can be divided into four quadrants. Anatomic studies have shown that screws placed
in the anterior superior and anterior inferior quadrants of the cup may injure the external iliac
vein and obturator artery, respectively. Posterior superior and posterior inferior placement (in
screws greater than 25 mm) may injure the sciatic nerve or the superior gluteal artery. The
common iliac artery is proximal to the acetabulum and is at least risk for injury from acetabular
screw placement.

Pellicci PM, Tria AJ Jr, Garvin KL (eds): Orthopaedic Knowledge Update: Hip and Knee Reconstruction 2. Rosemont, IL,
American Academy of Orthopaedic Surgeons, 2000, pp 207-215.

82. Figure 45 shows the lateral radiograph of a 19-year-old swimmer who has had back
pain for the past 2 months. What is the most likely diagnosis?

1- Disk degeneration
2- Limbus fracture
3- Degenerative retrolisthesis of L4-5
4- Spondylolysis
5- Osteoid osteoma

PREFERRED RESPONSE: 4

DISCUSSION: The patient has a pars interarticularis defect of L5 without apparent listhesis.
The other diagnoses are not present.

Papanicolaou N, Wilkinson RH, Emmans JB, Treves S, Micheli LJ: Bone scintigraphy and radiography in young athletes
with low back pain. Am J Roentgenol 1985;145:1039-1044.
Griffin LY (ed): Orthopaedic Knowledge Update: Sports Medicine. Rosemont, IL, American Academy of Orthopaedic
Surgeons, 1994, pp 291-302.

83. Figure 46 shows the AP radiograph of a


patient with right shoulder pain. What is
the most likely diagnosis?

1- Periosteal sleeve injury


2- Acute type 2 acromioclavicular joint separation
3- Rickets
4- Traumatic osteolysis
5- Distal clavicle fracture

Dr. Ahmed Altaei


American Academy of Orthopaedic Surgeons
2002 Anatomy Self-Assessment Examination
Page 47

PREFERRED RESPONSE: 4

DISCUSSION: Posttraumatic osteolysis of the distal portion of the clavicle is a condition that
can be a complication of acute or repetitive trauma. The distal end of the clavicle is frayed and
resorbed. Resorption may occur after weeks or months. The end of the clavicle may reconstitute
over a period of months, or the acromioclavicular joint may remain widened. The differential
diagnosis for distal clavicular erosion also includes rheumatoid arthritis, hyperparathyroidism,
neoplastic destruction, cleidocranial dysplasia, and pyknodysostosis. Acutely, a type 2
acromioclavicular joint injury does not result in erosion or resorption of the clavicle. Periosteal
sleeve injuries radiographically mimic acromioclavicular joint dislocation. Rickets occurs only
in childhood.

Cahill BR: Osteolysis of the distal part of the clavicle in male athletes. J Bone Joint Surg Am 1982;64:1053-1058.

84. The main arterial supply to the humeral head is provided by which of the following
arteries?

1- Anterior humeral circumflex


2- Posterior humeral circumflex
3- Thoracoacromial
4- Subscapular
5- Deep (profunda) brachial

PREFERRED RESPONSE: 1

DISCUSSION: The main arterial supply to the humeral head is provided by the ascending
branch of the anterior humeral circumflex artery and its intraosseous continuation, the arcuate
artery. There are significant intraosseous anastomoses between the arcuate artery, the posterior
humeral circumflex artery through vessels entering the posteromedial aspect of the proximal
humerus, the metaphyseal vessels, and the vessels of the greater and lesser tuberosities. Four-
part fractures and dissection during exposure affect perfusion of the humeral head.

Brooks CH, Revell WJ, Heatley FW: Vascularity of the humeral head after proximal humeral fractures: An anatomical
cadaver study. J Bone Joint Surg Br 1993;75:132-136.
Gerber C, Schneeberger AG, Vinh TS: The arterial vascularization of the humeral head: An anatomical study. J Bone Joint
Surg Am 1990;72:1486-1494.

85. Figure 47 shows a transverse MRI


scan of a patients left shoulder. The
findings reveal which of the following
abnormalities?

1- Subscapularis tear
2- Coracoid fracture
Dr. Ahmed Altaei
American Academy of Orthopaedic Surgeons
2002 Anatomy Self-Assessment Examination
Page 48
3- Osteonecrosis of the humeral head
4- Posterior labral tear
5- Hill-Sachs lesion

PREFERRED RESPONSE: 5

DISCUSSION: The MRI scan shows a defect in the posterior aspect of the humeral head,
commonly referred to as a Hill-Sachs lesion. This is an impaction fracture of the humeral head
that occurs during anterior shoulder dislocation. The abnormality on this image is an irregularity
of the posterior humeral head; the humeral head otherwise has a homogenous appearance. The
coracoid, subscapularis, and posterior labrum are normal.

Griffin LY (ed): Orthopaedic Knowledge Update: Sports Medicine. Rosemont, IL, American Academy of Orthopaedic
Surgeons, 1994, pp 47-63.
Cofield RH: Arthroscopy of the shoulder. Mayo Clin Proc 1983;58:501-508.

86. An 18-year-old man sustains an injury to the right brachial plexus after falling off
his bicycle. Examination reveals no rhomboideus major or minor muscle function.
This finding most likely indicates a preganglionic injury to which of the following
nerve roots?

1- C4
2- C5
3- C5 and C6
4- C7
5- C8 and T1

PREFERRED RESPONSE: 2

DISCUSSION: The rhomboideus major and minor muscles are innervated by the dorsal scapular
nerve, which is supplied entirely by the C5 nerve root. The dorsal scapular nerve arises just
distal to the dorsal root ganglion of the C5 nerve root. A functioning rhomboid muscle indicates
that an injury involving C5 nerve root fibers must be postganglionic or distal to the C5 dorsal
root ganglion.

Woodburne RT, Crelin ES, Kaplan FS, Dingle RV (eds): The Ciba Collection of Medical Illustrations. Summit, NJ, Ciba-
Geigy Corporation, 1987, vol 8, pp 23-28.
Millesi H: Brachial plexus injury in adults: Operative repair, in Gelberman RH (ed): Operative Nerve Repair and
Reconstruction. Philadelphia, PA, JB Lippincott, 1991, vol 2, pp 1285-1301.

87. A 53-year-old man with a history of severe left hip pain has a significant limp that is
the result of a 5-cm limb-length discrepancy. An AP radiograph is shown in Figure
48. The underlying etiology is most likely related to a history of

1- septic arthritis.

Dr. Ahmed Altaei


American Academy of Orthopaedic Surgeons
2002 Anatomy Self-Assessment Examination
Page 49
2- slipped capital femoral epiphysis.
3- femoral head fracture (Pipkin I).
4- developmental dysplasia of the hip.
5- Legg-Calve-Perthes disease.

PREFERRED RESPONSE: 5

DISCUSSION: Radiographic abnormalities such as coxa magna, coxa breva secondary to growth
arrest, and coxa plana and acetabular deformities are associated with healed Legg-Calve-Perthes
disease. Femoral heads that were flat yet congruent with the acetabulum are at risk for disabling
arthritis in the sixth decade of life in 50% of these untreated patients. As the normal ball-and-
socket joint deforms to a flattened cylinder, the hip loses abduction and rotation capability, while
retaining flexion and extension potential. If the femoral head is flat and is not concentric with
the acetabulum, early severe arthritis occurs. Hinge abduction and anterior impingement are
known sequelae of a flat, incongruent femoral head.

Beaty JH (ed): Orthopaedic Knowledge Update 6. Rosemont, IL, American Academy of Orthopedic Surgeons, 1999, pp 3-
23.

88. Where does the median nerve pass in the proximal forearm?

1- Through the pronator teres and deep to the flexor digitorum superficialis
2- Deep to the pronator teres and superficial to the flexor digitorum superficialis
3- Deep to the pronator teres and deep to the flexor digitorum superficialis
4- Adjacent to the ulnar artery
5- Superficial to the pronator teres and flexor digitorum superficialis

PREFERRED RESPONSE: 1

DISCUSSION: The median nerve passes through the pronator teres and deep to the flexor
digitorum superficialis. The ulnar artery passes deep to both.

Anderson JE (ed): Grants Atlas of Anatomy, ed 7. Baltimore, MD, Williams and Wilkins, 1978, pp 6-55.
Hoppenfeld S, deBoer P: Surgical Exposures in Orthopaedics: The Anatomic Approach. Philadelphia, PA, JB Lippincott,
1984, p 120.

Dr. Ahmed Altaei


American Academy of Orthopaedic Surgeons
2002 Anatomy Self-Assessment Examination
Page 50
89. The vascularity of the digital flexor tendons is significantly richer in what cross-
sectional region?

1- Volar ulnar quadrant


2- Volar radial quadrant
3- Peripheral one third
4- Dorsal one half
5- Center

PREFERRED RESPONSE: 4

DISCUSSION: The vascularity of the dorsal portion of the digital flexor tendons is considerably
richer than the volar portion. The other regions are not preferentially more vascular.

Hunter JM, Scheider LH, Makin EJ (eds): Tendon Surgery in the Hand. St Louis, MO, Mosby, 1987, pp 91-99.
Gelberman RH, Khabie V, Cahill CJ: The revascularization on healing flexor tendons in the digital sheath: A vascular
injection study in dogs. J Bone Joint Surg Am 1991;73:868-881.

90. Figures 49a and 49b show MRI scans of the shoulder. What is the most likely
diagnosis?

1- Rotator cuff tear


2- Normal anatomic variant
3- Stage II impingement
4- Bankart lesion
5- Acromioclavicular grade II sprain

PREFERRED RESPONSE: 1

DISCUSSION: The supraspinatus tendon shows clear detachment and retraction from its greater
tuberosity attachment by the absence of the normal dark subacromial signal extending to the
attachment on the greater tuberosity. There is no anterior inferior glenoid labral detachment that
usually is seen in a Bankart lesion. The acromioclavicular joint shows no evidence of separation.
The humeral head is migrated cranially, indicating a chronic rotator cuff tear.

Dr. Ahmed Altaei


American Academy of Orthopaedic Surgeons
2002 Anatomy Self-Assessment Examination
Page 51
Iannotti JP, Zlatkin MB, Esterhai JL, Kressel HY, Dalinka MK, Spindler KP: Magnetic resonance imaging of the shoulder:
Sensitivity, specificity, and predictive value. J Bone Joint Surg Am 1991;73:17-29.
Seeger LL, Gold RH, Bassett LW, Ellman H: Shoulder impingement syndrome: MR findings in 53 shoulders. Am J
Roentgenol 1988;150:343-347.
Williams MM, Snyder SJ, Buford D Jr: The Buford complex: The cord-like middle glenohumeral ligament and absent
anterosuperior labrum complex. A normal anatomic capsulolabral variant. Arthroscopy 1994;10:241-247.

91. A fracture of the radial head is surgically exposed using a posterolateral approach
to the elbow. Once the radial head is exposed, how should the arm be positioned to
best protect the posterior interosseous nerve from injury?

1- Full elbow flexion and wrist extension


2- Full forearm supination
3- Full elbow extension and wrist extension
4- Forearm pronation
5- Neutral forearm rotation

PREFERRED RESPONSE: 4

DISCUSSION: As long as the dissection stays proximal to the annular ligament, the posterior
interosseous nerve is not at risk for injury. However, to ensure that the nerve is as far removed
from the surgical field as possible, the forearm should be placed in pronation. Forearm
supination of any degree will bring the nerve toward the surgical field. A neutral position of the
forearm or elbow extension with wrist extension will not protect the posterior interosseous nerve.

Hoppenfeld S, deBoer P: Surgical Exposures in Orthopaedics: The Anatomic Approach, ed 2. Philadelphia, PA, Lippincott-
Raven, 1992, p 100.
Tubiana R, McCullough CJ, Masquelet AC: An Atlas of Surgical Exposures of the Upper Extremity. Philadelphia, PA, JB
Lippincott, 1990, p 106.

92. Figure 50 shows the MRI scan of a 20-year-old female college soccer player with
knee pain. What is the most likely diagnosis?

1- Normal knee
2- Posterior cruciate ligament tear
3- Quadriceps tendon rupture
4- Proximal tibia fracture
5- Femoral subchondral contusion

PREFERRED RESPONSE: 2

Dr. Ahmed Altaei


American Academy of Orthopaedic Surgeons
2002 Anatomy Self-Assessment Examination
Page 52
DISCUSSION: The MRI scan shows an acute complete tear of the posterior cruciate ligament.
No evidence is seen of a quadriceps tendon rupture, a tibia fracture, or a bone contusion.

Beaty JH (ed): Orthopaedic Knowledge Update 6. Rosemont, IL, American Academy of Orthopaedic Surgeons, 1999, pp
533-557.
Harner CD, Hoher J: Evaluation and treatment of posterior cruciate ligament injuries. Am J Sports Med 1998;26:471-482.

93. The tibiofibular overlap used to diagnose syndesmotic diastasis on an AP view is


most commonly measured between the

1- lateral border of the fibula and the medial border of the posterior tibial tubercle.
2- medial border of the fibula and the lateral border of the posterior tibial tubercle.
3- medial border of the fibula and the medial border of the deepest point of the
incisura fibularis.
4- medial border of the fibula and the lateral border of the anterior tibial tubercle.
5- medial border of the fibula and the lateral border of the deepest point of the
incisura fibularis.

PREFERRED RESPONSE: 4

DISCUSSION: The tibiofibular overlap is measured between the medial border of the fibula and
the lateral border of the anterior tibial tubercle. Plain radiographic assessment of the distal
tibiofibular syndesmosis requires AP and mortise views. The following criteria have been used
as the normal limits in adults: a talocrural angle of + or - 83 degrees with up to 5 degrees of
normal difference between both sides, a medial clear space of less than 4 mm, a talar tilt of less
than 2 mm, a tibiofibular clear space of less than 5 mm, a tibiofibular overlap of greater than or
equal to 0 mm, and a talar subluxation that is a subjective assessment of congruity of the tibial
articular surface and the talar dome; any incongruity is abnormal. It has been recommended to
obtain the first three measurements on the mortise view and the other three on the AP view.

Wuest TK: Injuries to the distal lower extremity syndesmosis. J Am Acad Orthop Surg 1997;5:172-181.
Stiehl JB: Ankle fractures with diastasis. Instr Course Lect 1990;39:95-103.

94. Figures 51a and 51b show subluxation of the

Dr. Ahmed Altaei


American Academy of Orthopaedic Surgeons
2002 Anatomy Self-Assessment Examination
Page 53
1- extensor carpi ulnaris.
2- distal radioulnar joint.
3- extensor carpi radialis longus.
4- extensor digiti minimi.
5- extensor pollicis longus.

PREFERRED RESPONSE: 1

DISCUSSION: The extensor carpi ulnaris tendon is shown subluxated from its tunnel at the
ulnar head; this requires disruption of the tendons subsheath.

Rowland SA: Acute traumatic subluxation of the extensor carpi ulnaris tendon at the wrist. J Hand Surg Am 1986;11:809-
811.
Green DP, Hotchkiss RN, Pederson WC (eds): Operative Hand Surgery, ed 4. New York, NY, Churchill Livingstone, 1999,
pp 1026-1027.

95. The so-called high ankle sprain from an external rotation mechanism of injury
typically involves injury to which of the following structures?

1- Posterior talofibular ligament


2- Deltoid ligament
3- Anterior inferior tibiofibular ligament
4- Calcaneofibular ligament
5- Extensor retinaculum

PREFERRED RESPONSE: 3

DISCUSSION: Ankle sprains most commonly involve injury to the lateral collateral ligaments of
the ankle (anterior talofibular, posterior talofibular, and calcaneofibular) from an inversion
mechanism of injury. A different entity has been more recently described that involves an
external rotation mechanism of injury that widens the ankle mortise and disrupts the anterior
inferior tibiofibular ligament. Deltoid ligament and extensor retinaculum injuries do occur,
although infrequently, and involve eversion and extreme plantar flexion mechanisms,
respectively.

Last RJ: Anatomy: Regional and Applied, ed 6. London, England, Churchill Livingstone, 1978, p 182.
Kaye RA: Stabilization of ankle syndesmosis injuries with a syndesmosis screw. Foot Ankle 1989;9:290-293.
Baxter DE: The Foot and Ankle in Sports. St Louis, MO, Mosby-Year Book, 1995, p 30.
Pfeffer GB (ed): Chronic Ankle Pain in the Athlete. Rosemont, IL, American Academy of Orthopaedic Surgeons, 2000, p
11.

96. In the first dorsal compartment of the wrist, what tendon most frequently contains
multiple slips?

1- Extensor pollicis longus


2- Extensor pollicis brevis

Dr. Ahmed Altaei


American Academy of Orthopaedic Surgeons
2002 Anatomy Self-Assessment Examination
Page 54
3- Extensor carpi radialis longus
4- Extensor carpi radialis brevis
5- Abductor pollicis longus

PREFERRED RESPONSE: 5

DISCUSSION: The first extensor compartment of the wrist typically contains a single extensor
pollicis brevis tendon and the abductor pollicis longus tendon that nearly always has multiple
tendon slips. The extensor pollicis brevis tendon is frequently found to be separated from the
slips of the abductor pollicis longus tendon by an intracompartmental septum. During surgery,
this septum must be divided to complete the release of the compartment.

Jackson WT, Viegas SF, Coon TM, Stimpson KD, Frogameni AD, Simpson JM: Anatomical variations in the first extensor
compartment of the wrist: A clinical and anatomical study. J Bone Joint Surg Am 1986;68:923-926.
Minamikawa Y, Peimer CA, Cox WL, Sherwin FS: DeQuervains syndrome: Surgical and anatomical studies of the fibro-
osseous canal. Orthopedics 1991;14:545-549.

97. The preferred surgical approach to the elbow of a child with an irreducible type III
supracondylar distal humerus fracture and pulseless extremity is through which of
the following muscle intervals?

1- Pronator teres and the brachialis


2- Pronator teres and the triceps
3- Pronator teres and the biceps
4- Brachioradialis and the biceps
5- Brachioradialis and the brachialis

PREFERRED RESPONSE: 1

DISCUSSION: In a type III supracondylar distal humerus fracture of the elbow, the brachial
artery can become incarcerated, yielding a pulseless extremity. In this situation, closed reduction
may not be effective; therefore, open management is often necessary. The preferred surgical
approach to the brachial artery and to this fracture is the anterior approach to the cubital fossa.
The lacertus fibrosis is incised, and the dissection is carried out between the brachialis
(musculocutaneous nerve) and the pronator teres (median nerve), mobilizing the brachial artery.
Once the brachial artery is mobilized, the anterior elbow joint capsule may be exposed. The
interval between the brachialis and the biceps describes the anterolateral approach to the elbow
more commonly used for exposure of the proximal aspect of the posterior interosseous nerve.
The dissection interval between the brachioradialis and the pronator teres describes the proximal
extent of the anterior approach to the radius.

Tubiana R, McCullough CJ, Masquelet AC: An Atlas of Surgical Exposures of the Upper Extremity. Philadelphia, PA, JB
Lippincott, 1990, p 115.
Hoppenfeld S, deBoer P: Surgical Exposures in Orthopaedics: The Anatomic Approach, ed 2. Philadelphia, PA, Lippincott-
Raven, 1992, p 119.

Dr. Ahmed Altaei


American Academy of Orthopaedic Surgeons
2002 Anatomy Self-Assessment Examination
Page 55
98. A 48-year-old man has recurrent right knee pain. Figure 52a shows the sagittal
proton density T2-weighted MRI scan, and Figure 52b shows the sagittal T2-
weighted MRI scan at the same level. The arrow is pointing to a

1- popliteal cyst.
2- posterior cruciate ligament tear.
3- torn and displaced posterior horn of the medial meniscus.
4- normal meniscofemoral ligament of Humphry.
5- normal meniscofemoral ligament of Wrisberg.

PREFERRED RESPONSE: 3

DISCUSSION: Meniscal tears have many configurations and locations. The normal medial
meniscus has a bow-tie configuration on the two most medial consecutive sagittal views.
Toward the center of the joint the anterior and posterior horns have a triangular shape. These
images show an abnormal intra-articular low-signal structure located anterior to the intact
posterior cruciate ligament. This most likely represents a torn and displaced posterior horn of the
medial meniscus, sometimes called double PCL sign. A popliteal cyst and ligaments of
Wrisberg and Humphry are not visible on these figures.

Helms CA: MR image of the knee, in Fundamentals of Skeletal Radiology, ed 2. Philadelphia, PA, WB Saunders, 1995, pp
172-191.
Mink JH, Deutsch AL: The knee, in MRI of the Musculoskeletal System, ed 1. New York, NY, Raven Press, 1990, pp 251-
387.

99. Figure 53 shows a thoracolumbar specimen as


viewed from posterior to anterior following removal
of all posterior elements. Which of the following
structures does the red string pass under?

1- Diskovertebral ligament
2- Posterior longitudinal ligament
3- Cauda equina

Dr. Ahmed Altaei


American Academy of Orthopaedic Surgeons
2002 Anatomy Self-Assessment Examination
Page 56
4- Batsons plexus
5- Ligamentum flavum

PREFERRED RESPONSE: 5

DISCUSSION: The string passes under the ligamentum flavum as it runs from the posterior
aspect of the vertebra above to the inferior aspect of the vertebra below in the sagittal midline.
This is an important structure in diskectomy and in posterior approaches to the thoracolumbar
spine and neural elements. It is rarely visualized in its entirety because typical exposures provide
only a limited view.

Frymoyer JW (ed): The Adult Spine: Principles and Practice. New York, NY, Raven Press, 1991, vol 2, p 1465.

100. A 5-year-old girl sustained a comminuted Salter-Harris type IV fracture of the left
distal tibia 2 years ago. The AP radiograph shown in Figure 54a reveals a growth
arrest and a 1.4-cm limb-length discrepancy. The ankle is in approximately 20
degrees of varus. Figure 54b shows a coronal reconstruction image of the distal
tibial physis, and Figure 54c shows a sagittal reconstruction image of the same area.
On the sagittal reconstruction image, the bar extends from the 9-mm mark to the
24-mm mark in 3-mm increments. On the coronal image, the bar extends from the
9-mm mark to the 24-mm mark, also in 3-mm increments. A map of the physeal
bar based on these measurements is shown in Figure 54d. Initial treatment should
consist of

Dr. Ahmed Altaei


American Academy of Orthopaedic Surgeons
2002 Anatomy Self-Assessment Examination
Page 57
1- bony bar resection and distal fibula
epiphysiodesis.
2- bony bar resection and corrective osteotomy.
3- bony bar resection and physiodesis of the
opposite distal tibial physis.
4- corrective osteotomy and a limb-lengthening
procedure.
5- corrective osteotomy and physiodesis of the
opposite distal tibial physis.

PREFERRED RESPONSE: 2

DISCUSSION: Mapping of a physeal bar from biplane polytomography or CT helps to identify


lesions that should be treated surgically and aids in planning the surgical approach and resection.
Criteria for surgical excision are at least 2 years of longitudinal growth remaining and
involvement of no more than 50% of the physis. Osteotomy is required if angular deformity is
greater than 20 degrees. Although this physeal bar is large, it is slightly less than 50% of the
total area of the physis. Limb lengthening in this case should be reserved for failure of bar
resection. Physiodesis of the opposite distal tibia at this age would result in disproportionate
shortening of both tibiae.

Carlson WO, Wenger DR: A mapping method to prepare for surgical excision of a partial physeal arrest. J Pediatr Orthop
1984;4:232-238.
Peterson HA: Partial growth plate arrest and its treatment. J Pediatr Orthop 1984;4:246-258.

Dr. Ahmed Altaei

Вам также может понравиться